Anda di halaman 1dari 113

Manual de Matemtica para o 12 ano Matemtica A

NIUaleph 12
LIVRO DE EXERCCIOS

VOLUME 1
Jaime Carvalho e Silva Joaquim Pinto Vladimiro Machado 2012

Ttulo
NiuAleph 12 - Livro de Exerccios para o 12. ano de Matemtica A

Autores
Jaime Carvalho e Silva (Editor) Joaquim Pinto Vladimiro Machado

Capa e Design
Elisa Silva

Conceo Tcnica
Vtor Teodoro Joo Fernandes

Colaborao
Antnio Marques do Amaral, Raul Gonalves e Sofia Marques

Imagens e fontes
As imagens utilizadas neste manual pertencem ao domnio pblico ou, nas situaes indicadas, aos respetivos autores, sob as Licenas Creative Commons Attribution-ShareAlike 3.0 http://creativecommons.org/licenses/by-sa/3.0/) ou Creative Commons Attribution 3.0 http://creativecommons.org/licenses/by/3.0/

As fontes utilizadas neste manual pertencem s famlias Latin Modern e Latin Modern Math, desenvolvidas pela GUST http://www.gust.org.pl/projects/e-foundry/lm-math/index_html

ISBN
978-989-97839-1-1

Edio
1. edio/verso 1

Data
2012 Este ficheiro de distribuio livre mas os direitos permanecem com os respetivos autores. No permitida a impresso deste ficheiro.

ndice geral
Volume 1
(Captulos 1 a 8) Exerccios globais de 2. oportunidade Recomendaes do GAVE Testes de tempo limitado Solues Sntese

Volume 2
(Captulos 9 a 17) Exerccios globais de 2. oportunidade Recomendaes do GAVE Testes de tempo limitado Solues Sntese

ndice
Introduo 6

Exerccios globais de 2. oportunidade


Captulo 1 - possvel? provvel? Captulo 2 - Probabilidades Captulo 3 - Probabilidade condicionada Captulo 4 - Distribuio de probabilidades Captulo 5 Anlise Combinatria Captulo 6 - Tringulo de Pascal e Binmio de Newton Captulo 7 Funo exponencial Captulo 8 Funo logartmica

9
9 13 17 21 24 27 29 33

Recomendaes do GAVE
Captulo 1 - Resoluo de problemas da vida real Tarefas resolvidas Tarefas propostas Questes de escolha mltipla Captulo 2 - Problemas que envolvem clculos mais elaborados no conjunto dos nmeros reais Tarefas resolvidas Tarefas propostas Questes de escolha mltipla Captulo 4 - Exerccios que pressupem raciocnios demonstrativos Tarefas resolvidas Tarefas propostas Captulo 5 - Utilizar a calculadora grfica para resolver problemas Tarefas resolvidas

37
39 39 47 53 56 56 60 61 63 63 65 66 66

Tarefas propostas

70

Testes de tempo limitado


Teste 1 Probabilidades Escolha mltipla Teste 2 Probabilidades Escolha mltipla Teste 3 Probabilidades Itens de resposta aberta Teste 4 Probabilidades Itens de resposta aberta Teste 5 Probabilidades

73
73 77 80 81 85

Solues

89

Sntese

108

0.

Introduo

Para qu fazer exerccios?


Como bem chamou a ateno o matemtico Ian Stewart, grande investigador matemtico da Universidade de Warwick (Inglaterra) e divulgador da matemtica, com mais de 80 livros publicados, Os problemas so a fora motriz da Matemtica Ento espera-se que os alunos resolvam problemas. Estudar matemtica implica resolver problemas. Uns mais simples podero ser chamados exerccios, outros mais extensos ou complexos podero ser chamados tarefas. No se preocupem com estas designaes que existem mais para organizar as coisas do que verdadeiramente para classificar os problemas.

Quantos exerccios devo fazer?


Saber quantos exerccios resolver ou que tipo de exerccios resolver um dos dilemas mais comuns dos estudantes. So frequentes perguntas como: Como fao isso professor? Qual a frmula que se usa? Que conta temos que fazer? O senhor no ensinou isso! No h milagres e na pgina interior da contracapa deste livro aparecem os conselhos de um grande matemtico hngaro George Polya (18881985), que se dedicou reflexo sobre os mtodos de resoluo de problemas em todos os nveis de ensino. Um outro matemtico, o australiano Terence Tao, que em 2006 ganhou a medalha Fields (tambm chamado o Prmio Nobel da Matemtica) descreve assim o seu mtodo de resolver problemas: Hoje, comigo, sempre assim: Vamos tentar esta ideia. Isso leva-me a algum progresso, ou ento no funciona. Agora tentemos aquilo. Oh, h aqui um pequeno atalho. Trabalhamos durante tempo suiciente e, a certa altura, conseguimos progredir num problema difcil entrando pela porta das traseiras. No inal, o que normalmente acontece : Olha, resolvi o problema. O matemtico espanhol Miguel de Guzmn (19362004), autor de livros de divulgao como Aventuras Matemticas e Contos com contas, dava como primeiro conselho o seguinte: Antes de fazer tenta entender efetivamente fundamental que se leia com ateno o enunciado do problema e se tente entender bem o que dado e o que pedido. Um minuto perdido na leitura do enunciado pode salvar 30 minutos de resoluo intil porque no se responde realmente ao que pedido. O grande matemtico portugus Sebastio e Silva (19141972) preocupava-se com a resoluo de problemas sem cuidados na sua escolha. Escreveu: preciso combater o excesso de exerccios que, como um cancro, acaba por des6
Introduo

truir o que pode haver de nobre e vital no ensino. preciso evitar certos exerccios artiiciosos ou complicados, especialmente em assuntos simples.(...) mais importante reletir sobre o mesmo exerccio que tenha interesse, do que resolver vrios exerccios diferentes, que no tenham interesse nenhum.(...) Entre os exerccios que podem ter mais interesse iguram aqueles que se aplicam a situaes reais, concretas. Neste livro de exerccios os autores tiveram a preocupao de selecionar cuidadosamente os exerccios pelo seu interesse e no apenas para fazerem nmero de pginas. Primeiro aparecem o que chamamos exerccios de 2 oportunidade, ou seja, exerccios que devem ser feitos apenas depois de resolvidos os exerccios do manual escolar e apenas em caso de necessidade. Se no conseguiste dominar alguma parte da matria, se queres refrescar a tua mente com uma matria que tens medo de j ter esquecido, se queres testar o teu prprio conhecimento, pega nestes exerccios, respeitando o grau de dificuldade (se dominas bem os exerccios simples de determinado captulo no precisas de fazer mais exerccios fceis). Depois aparecem os exerccios de matrias que o GAVE descobriu que so aquelas onde os alunos tm mais dificuldades e a que chamamos Recomendaes do GAVE. Esta parte contm algumas tarefas resolvidas que deves tentar resolver por ti; s depois de tentares resolver cada tarefa que deves olhar para a respetiva resoluo e tentar compreend-la. No te esqueas que cada problema pode ter vrios processos igualmente vlidos de resoluo, como se pode ver bem no caso da Tarefa 5. Na terceira parte preparmos testes de tempo limitado, de 45m e 90m, com uso de calculadora e sem uso de calculadora, para conseguires testar a tua capacidade de resolver um certo nmero de exerccios dentro de um intervalo temporal fixado previamente. Este um aspeto que tambm os relatrios do GAVE identificam como os alunos tendo dificuldade.

Como detetar alguns erros mais comuns


Na pressa da resoluo de um problema comum cometerem-se erros que podem estragar completamente um problema. Por exemplo: preciso usar muitas frmulas e por vezes trocam-se uns sinais na frmula ou usa-se a frmula ao contrrio. Como ter a certeza que a frmula est correta? Quais os principais cuidados a ter? Havendo dvidas quanto validade de determinada frmula, o melhor testar a frmula com casos particulares. Por exemplo, a expresso no pode ser igual expresso porque se fizermos , a primeira expresso vale e a segunda vale zero e no podem assim ser iguais para todos os valores de x e y se nem sequer o so para valores particulares de x e de y. Outra estratgia til usar a calculadora grfica ou o computador para traar um grfico, mesmo quando no conseguimos obter valores exatos. Por exemplo, se tivermos dvidas se o ponto (1,1) satisfaz simultaneamente as desigualdades

Introduo

poderemos recorrer calculadora grfica para obter o grfico seguinte

e concluir que tal ponto, no estando na regio sombreada, no satisfaz simultaneamente as duas desigualdades dadas. Podemos ter de provar isso analiticamente mas j ficamos a saber a resposta o que ajuda na resoluo e permite controlar eventuais erros de clculo. Um modo de controlar se duas funes so realmente inversas usar uma calculadora ou computador e procurar o grfico da respetiva composta. Por exemplo, para as funes e se tentarmos traar o grfico de

obteremos a funo identidade. No prova nada, mas permite verificar a nossa ideia (ou detetar um erro se no obtivermos a funo identidade).

Outros conselhos poderiam ser avanados, mas ficaro para o segundo volume. Ao longo do ano escolar os autores iro disponibilizando na internet, na pgina
HTTP:// NIUALEPH.EU

mais tarefas e desafios e provas globais para tu poderes ir encontrando desafios sempre novos.

Bom trabalho!

Introduo

1.
Pratica
1.

Exerccios globais de 2. oportunidade


C1

CAPTULO 1

POSSVEL?

PROVVEL?

Quando se fazem previses sobre um acontecimento, utilizam-se com frequncia frases como: quase certo, bastante provvel, pouco provvel, quase impossvel. Associa uma destas frases s seguintes previses sobre o clima na cidade de Faro no dia 15 de Agosto: 1.1 1.2 1.3 1.4 1.5 Nevar. Chover. A temperatura mxima ser superior a 20. O cu estar limpo. O Sol brilhar mais de 3 horas.

2.

Observa a roda da sorte da igura. Considera a experincia: rodar o ponteiro e anotar o nmero que sai. 2.1 2.2 Indica o espao de resultados. Indica o subconjunto do espao de resultados associado a cada um dos seguintes acontecimentos. 2.2.1 2.2.2 2.2.3 2.2.4 2.2.5 2.2.6 2.2.7 2.2.8 Sair nmero mpar. Sair nmero fatorizvel. Sair mltiplo de 3. Sair 2 ou 3. Sair 9. No sair 9. Sair 11, 13 ou 15. No sair 11, nem 13, nem 15. 11 10 9 8 7 6 5 12 1 2 3 4

Exerccios globais de 2. oportunidade

2.3

Considera os acontecimentos:

A: Sair nmero par. B: Sair nmero maior ou igual a 3. Utilizando apenas estes dois acontecimentos e as operaes de interseo, reunio e complementao, caracteriza os seguintes acontecimentos: 2.3.1 2.3.2 2.3.3 2.3.4 3. Sair nmero mpar. Sair nmero 1. Sair 2 ou sair um nmero mpar. Sair nmero par menor do que 3.

Considera a experincia que consiste na extrao de uma carta de um baralho de 52 cartas e os acontecimentos: A: Sair copas B: Sair valete C: Sair 10 de capas ou de ouros 3.1 3.2 Indica qual o espao de resultados associado a esta experincia. Traduz por palavras o signiicado dos seguintes acontecimentos: , , , . , ,

4.

Considera a experincia aleatria que consiste em veriicar o sexo dos ilhos das famlias de trs ilhos. 4.1 4.2 4.3 Indica qual o espao de resultados associado a esta experincia. Considera o acontecimento pelo menos um dos ilhos do sexo masculino. Quantas ocorrncias pode ter este acontecimento (nmero de elementos do acontecimento)? Representa por um diagrama de Venn o acontecimento da alnea anterior.

5.

Lanamos dois dados no cbicos de cores diferentes numerados de 1 a 9 e tomamos nota dos resultados das faces superiores. Determina: 5.1 5.2 5.3 O espao de resultados. O acontecimento obter pelo menos um 5. O acontecimento obter pelo menos um resultado superior a 7.

6.

Uma equipa de basquetebol de Lamego e outra de Viseu esto na inal de uma competio nacional em que o vencedor a primeira equipa que ganhar 3 jogos. A equipa de Lamego ganhou o primeiro jogo. Qual o espao de resultados?

10

Exerccios globais de 2. oportunidade

7.

Lanamos dois dados no cbicos de cores diferentes numerados de 1 a 9 e tomamos nota dos resultados das faces superiores. Determina o acontecimento contrrio do acontecimento Sair face par.

Pensa e resolve
8. Lanamos dois dados no cbicos de cores diferentes numerados de 1 a 9 e tomamos nota dos resultados das faces superiores. D um exemplo de: 8.1 8.2 8.3 9. Um acontecimento elementar. Um acontecimento certo. Um acontecimento impossvel.

No lanamento de um dado cbico comum, consideremos os acontecimentos: A: sair face par B: sair face menor que 3 9.1 Deine em extenso o acontecimento contrrio de: 9.1.1 9.1.2 9.1.3 9.1.4 B A

10.

De uma urna que contm duas bolas amarelas e duas bolas roxas, retira-se uma bola ao acaso e regista-se a cor. 10.1 10.2 10.3 Qual o espao de resultados? Quais os acontecimentos elementares? Considera os seguintes acontecimentos: A: Sair bola amarela B: Sair bola vermelha C: No sair bola roxa D: No sair bola amarela nem roxa 10.3.1 Representa os acontecimentos por conjuntos. 10.3.2 Indica um acontecimento certo e um acontecimento impossvel.

Exerccios globais de 2. oportunidade

11

11.

No lanamento de um dado, consideremos os acontecimentos: A: sair face par e B: sair face menor que 3. Deine em extenso o acontecimento contrrio de B \ A .

Relete
12. Para cada uma das seguintes airmaes, indica quais so verdadeiras e quais so falsas: 12.1 12.2 12.3 12.4 12.5 12.6 13. Numa experincia aleatria pode no haver acontecimento certo. Numa experincia aleatria pode no haver acontecimento impossvel. O acontecimento contrrio de um acontecimento certo sempre impossvel. O acontecimento contrrio de um acontecimento elementar sempre impossvel. O acontecimento contrrio do acontecimento contrrio de um acontecimento elementar sempre impossvel. O acontecimento contrrio do acontecimento contrrio de um acontecimento impossvel sempre impossvel.

Num espao S, considera dois acontecimentos A e B diferentes, e supe que nenhum deles impossvel ou certo. Explica quando se poder ter que impossvel.

12

Exerccios globais de 2. oportunidade

C2 CAPTULO 2 PROBABILIDADES
Pratica
1. Lanou-se uma moeda de euro ao ar duas vezes seguidas. Uma moeda de euro tem uma face europeia e uma face nacional. Calcula a probabilidade de obter duas faces europeias no lanamento. Lanou-se uma moeda de euro ao ar trs vezes seguidas. Calcula a probabilidade de obter trs faces europeias no lanamento. Lanou-se uma moeda de euro ao ar quatro vezes seguidas. 3.1 3.2 4. Calcula a probabilidade de obter: trs faces europeias e uma nacional no lanamento. Pelo menos duas faces europeias.

2. 3.

Num saco h 5 bolas vermelhas, 3 azuis e 2 verdes. Retiram-se sucessivamente do saco trs bolas, sem repor nenhuma. Determina: 4.1 4.2 4.3 A probabilidade de sarem as 3 azuis. A probabilidade de sarem 3 bolas da mesma cor. A probabilidade de sarem 3 bolas de 3 cores diferentes. e :

5.

Seja S o conjunto de resultados associados a uma certa experincia aleatria. Se A e B so os acontecimentos apresentados a seguir, determina em cada caso 5.1 5.2 5.3 , , , , , ,

6.

Lanou-se ao ar um dado tetradrico no equilibrado com as faces numeradas de 1 a 4. Depois de 1000 lanamentos, obtiveram-se os seguintes valores para as probabilidades de 3 das faces: P({1}) = 0,6, P({2}) = 0,18 e P({3}) = 0,21. Qual a probabilidade de sair a face com o nmero 4? Enuncia uma axiomtica para as probabilidades. Prova que quaisquer que sejam os acontecimentos A e B, .

7.

Exerccios globais de 2. oportunidade

13

one Euro coins por Images Money, http://www.lickr.com/photos/59937401@N07/5929570123/

8.

Seja S o conjunto de resultados (com um nmero inito de elementos) associado a uma certa experincia aleatria. Sejam A e B dois acontecimentos, contidos em S, nenhum deles impossvel, nem certo. Para cada alnea procura exemplos concretos para S, A e B de tal modo que se veriique que 8.1 8.2 8.3 8.4

9.

Lana-se um dado equilibrado de 8 faces com as faces numeradas de 1 a 8. Considera os acontecimentos: A: sair face mpar B: sair face de nmero maior ou igual a 4 Determina o acontecimento contrrio de com o seu acontecimento contrrio? . Qual a probabilidade da unio de

10.

Lanam-se dois dados no viciados, um octadrico com as faces numeradas de 1 a 8 e outro dodecadrico com as faces numeradas de 1 a 12. Determina a probabilidade de: 10.1 10.2 Sair um nmero diferente em ambos os dados. Sair um nmero igual em ambos os dados.

Pensa e resolve
11. Por vezes, mais fcil determinar a probabilidade do acontecimento contrrio ao que pedido por envolver uma contagem mais fcil. Aplica este princpio seguinte situao: Lanam-se dois dados cbicos equilibrados, tendo ambos as faces numeradas de 1 a 6. Qual a probabilidade de a soma das pintas obtidas ser inferior ou igual a 10. Seja S o conjunto de resultados (com um nmero inito de elementos) associado a uma certa experincia aleatria. Sejam A e B dois acontecimentos, contidos em S, nenhum deles impossvel, nem certo. Para cada alnea procura exemplos concretos para S, A e B, se existirem, de tal modo que no se verifique que: 12.1 12.2 12.3 12.4

12.

14

Exerccios globais de 2. oportunidade

13.

Seja S o conjunto de resultados associado a uma certa experincia aleatria. Sejam A e B dois acontecimentos tais que e . , e . Calcula ,

14.

Um jogador utiliza um dado cbico no equilibrado, com as faces numeradas de 1 a 6. A probabilidade de sair cada uma das 5 primeiras faces dada pela tabela seguinte: Nmero Probabilidade 14.1 14.2 1 0,1 2 0,2 3 0,1 4 0,15 5 0,15 6 ?

Determina o valor em falta. Determina a probabilidade de: 14.2.1 Sair nmero par. 14.2.2 Sair um nmero inferior ou igual a 3. 14.2.3 Sair o nmero 6.

15. 16.

Dois acontecimentos dizem-se incompatveis se a realizao de um deles implica a no realizao do outro. Exprime este conceito usando conjuntos. Mostra que se A e B so dois acontecimentos se tem .

Relete
17. O Joo e a Maria vo jogar aos dados com as seguintes regras: Um dado cbico equilibrado com as faces numeradas de 1 a 6 lanado ao ar duas vezes. O Joo ganha se sair pelo menos um 1 ou um 6. A Maria ganha se sarem dois nmeros pares. A questo que se coloca : este jogo equitativo, isto , tanto o Joo como a Maria tm igual probabilidade de ganhar? 18. Num jogo de dados so lanados dois dados comuns e se a soma das pintas dos dados for estritamente superior a 7 ento tu ganhas o jogo. Caso contrrio o teu adversrio que ganha. Quem favorecido neste jogo?

Exerccios globais de 2. oportunidade

15

19.

Diz se as airmaes seguintes so verdadeiras ou falsas: 19.1 Se A e B so acontecimentos em S, o conjunto de resultados associado a uma certa experincia aleatria, ento 19.2 sempre superior a . sempre superior a .

19.3 19.4 19.5 20.

possvel ter possvel ter

, ,

,e ,e

. .

Dois acontecimentos incompatveis so contrrios.

Em 2011, em Portugal, estavam matriculados no Ensino Superior 396 268 indivduos e desses 28 657 estudavam Cincias, Matemtica e Informtica-CMA (fonte: Pordata). Destes estudantes, 46,6% eram do sexo feminino. 20.1 Reproduz no teu caderno e completa a tabela seguinte: No CMA Feminino Masculino 28 657 20.2 396 268 CMA total

Escolhemos, ao acaso, um estudante matriculado no Ensino Superior em 2011. Considera os seguintes acontecimentos: A: um estudante de Cincias, Matemtica e Informtica-CMA B: do sexo feminino C: Estuda Cincias, Matemtica e Informtica-CMA e do sexo feminino D: do sexo masculino e no estuda Cincias, Matemtica e Informtica-CMA Calcula a probabilidade de cada um destes acontecimentos. Arredonda o resultado s centsimas.

20.3 20.4

Os acontecimentos A e D so incompatveis? Considera o acontecimento . Deine por meio de uma s frase este acontecimento e calcula a sua probabilidade. Arredonda o resultado s centsimas.
(adaptado do exame do 12. ano, Frana, 1997)

16

Exerccios globais de 2. oportunidade

C3 CAPTULO 3 PROBABILIDADE
Pratica
1. 2. 3. Uma urna contm cinco bolas brancas e doze pretas, equiprovveis. Ao extrair duas bolas qual a probabilidade de que eles sejam da mesma cor? Calcula a probabilidade de a soma das faces de dois dados ser maior que 10 sabendo que no primeiro dado saiu um seis. Seja S o espao de resultados associado a uma certa experincia aleatria. Sejam A e B dois acontecimentos tais que P(A) = 43% , P(B ) = 77% e P(A B ) = 82% . Usando um diagrama de Venn determina o valor das probabilidades condicionadas: 3.1 3.2 4. P(A | B ) P(B | A)

CONDICIONADA

Seja S o espao de resultados associado a uma certa experincia aleatria. Sejam A e B dois 4 Venn determina o valor das probabilidades: 4.1 4.2 P(A B ) P(B | A) acontecimentos tais que P(A) = 1 , P(B ) = 1 3 e P(A | B ) = 1 9 . Usando um diagrama de

5.

Numa turma de 12. ano sabe-se que a probabilidade de um aluno ter dvidas a matemtica de 55%, de ter dvidas a portugus de 30% e de ter simultaneamente dvidas a ambas a disciplinas de 20%. Calcula, apresentando o resultado na forma de frao irredutvel, a probabilidade de um aluno: 5.1 5.2 5.3 5.4 5.5 5.6 Ter dvidas a matemtica sabendo que tem dvidas a portugus. Ter dvidas a portugus sabendo que tem dvidas a matemtica. Ter dvidas a matemtica sabendo eu no tem dvidas a portugus. Ter duvidas a portugus sabendo que no tem dvidas a matemtica. No ter dvidas a matemtica sabendo que tem dvidas a portugus. No ter dvidas a portugus sabendo que no tem dvidas a matemtica.

Exerccios globais de 2. oportunidade

17

6.

Na Escola Secundria Anastcio da Cunha, foi feito um inqurito sobre a leitura de 3 revistas de desportos motorizados: AutoRpido, BoaCorrida e CorreRpido. Dos 100 alunos interrogados, 57 lem AutoRpido, 42 lem BoaCorrida, 38 lem CorreRpido, 22 lem AutoRpido e BoaCorrida, 14 lem BoaCorrida e CorreRpido, 16 lem AutoRpido e CorreRpido, 8 lem AutoRpido, BoaCorrida e CorreRpido. Usando um diagrama de Venn, calcula o nmero de alunos que: 6.1 6.2 6.3 6.4 6.5 Lm apenas AutoRpido e BoaCorrida. Lm apenas BoaCorrida e CorreRpido. Lm apenas BoaCorrida. Lm apenas CorreRpido. No lm nenhuma das trs revistas.

7.

Suponhamos que na Escola Secundria Lus de Albuquerque foram inquiridos 300 alunos dos dois sexos sobre as suas preferncias de leitura de jornais dirios entre o NoticiasFrescas e o TodaAVerdade. Obtiveram-se os seguintes resultados: L NoticiasFrescas Rapazes Raparigas 7.1 7.2 120 20 L TodaAVerdade 80 80

Suponhamos que se selecionou um aluno ao acaso. Qual a probabilidade de ler NoticiasFrescas sabendo que Rapariga? Suponhamos que se selecionou um aluno ao acaso. Qual a probabilidade de ser Rapariga sabendo que l TodaAVerdade?

8.

Suponhamos que num saco h 3 bolas vermelhas e 2 bolas azuis. Das bolas vermelhas 2 so redondas e uma triangular. Das bolas azuis 1 redonda e 1 triangular. Retira-se ao acaso uma pea do saco. Qual a probabilidade de ser redonda sabendo que azul? Lana-se um dado equilibrado, com as faces numeradas de 1 a 6, duas vezes consecutivas. Determina a probabilidade de no primeiro lanamento ter sado a face com o nmero 1, sabendo que a soma dos nmeros sados 4. Lanam-se dois dados. 10.1 10.2 Qual a probabilidade de obter uma soma igual a 7? Sabendo que a soma 7, qual a probabilidade de que em algum dos dados tenha sado um 3?

9.

10.

11. 12.

Numa experincia aleatria os acontecimentos A e B so tais que P(A) = 0,12 e P(B) = 0,90. Os acontecimentos so independentes? Numa experincia aleatria os acontecimentos A, B e C so tais que P(A) = 1/2 , P(B) = 1/3 e P(C) = 1/4. Os acontecimentos so independentes?

18

Exerccios globais de 2. oportunidade

Pensa e Resolve
13. O daltonismo est associado a uma alterao gentica que mais frequente nos homens que nas mulheres. Um estudo feito em larga escala revela que: Daltnico Homens Mulheres total 8,1% 0,5% 8,6% No daltnico 45% 46,4% 91,4% total 53,1% 46,9% 100%

Determina a probabilidade de: 13.1 13.2 13.3 13.4 Sabendo que homem ser daltnico. Sabendo que mulher ser daltnica. Sabendo que daltnico sabendo que homem. Sabendo que daltnico sabendo que mulher.

14.

Numa companhia rea a probabilidade de um voo partir dentro do horrio previsto de 83%, a probabilidade de chegar no horrio previsto de 82% e a probabilidade de que o voo parta e chegue no horrio previsto de 78%. Calcula: 14.1 14.2 14.3 A probabilidade do voo chegar no horrio previsto tendo sado no horrio previsto. A probabilidade do voo ter sado no horrio sabendo que chegou no horrio previsto. A probabilidade de no chegar no horrio previsto sabendo que no saiu no horrio previsto.

15. 16.

Se a probabilidade de nascer um rapaz de 0,51 e de nascer uma rapariga de 0,49, determina a probabilidade de que dois gmeos sejam do mesmo sexo. Na sequncia da descoberta na Artilndia de um primeiro caso de uma doena contagiosa no mortal, o Governo desse pas promoveu uma importante campanha de vacinao. Em consequncia 70% dos habitantes foram vacinados. Um estudo feito mais tarde revelou que 5% dos vacinados foram atingidos em diversos graus pela doena, percentagem que se elevou a 60% nos no vacinados. 16.1 16.2 Determina a probabilidade de um indivduo escolhido ao acaso na populao da Artilndia ter sido atingido pela doena. Calcula a probabilidade de um indivduo ter sido vacinado, sabendo que foi atingido pela doena.

17.

Mostra que se dois acontecimentos so independentes ento os seus contrrios tambm so independentes. 19

Exerccios globais de 2. oportunidade

18.

Seja S o conjunto de resultados associado a uma experincia aleatria. Sejam X e Y dois acontecimentos possveis e incompatveis. Prova que

Relete
19. O facto de ser surdo independente de ser do sexo masculino ou feminino, tendo em considerao isso calcula as quatro probabilidades que faltam na tabela seguinte: Surdo Masculino Feminino total 20. 0,004 0,996 No surdo total 0,531 0,469 1,000 e

Se dois acontecimentos A e B so independentes pode acontecer que ?

21.

De dois acontecimentos A e B sabemos que

. Determina

para que os acontecimento A e B sejam independentes. 22. Seja S o conjunto de resultados associado a uma experincia aleatria. Sejam A, B dois acontecimentos possveis. Sabe-se que: P (A B ) = P (B ) . Ser que se pode airmar que 23. ?

Mostra que o acontecimento impossvel independente de qualquer outro acontecimento.

20

Exerccios globais de 2. oportunidade

C4 CAPTULO 4 DISTRIBUIO
Pratica
1. A distribuio de probabilidade de uma dada varivel aleatria 1 0,1 Determina 1.1 1.2 1.3 2. Lana-se duas vezes um dado equilibrado, com as faces numeradas de 1 a 6. Seja X o nmero de vezes que sai a face 6 nos dois lanamentos. Qual a distribuio de probabilidades da varivel aleatria X? O grico representado de uma distribuio normal.
y
1,0

DE PROBABILIDADES

2 0,1

3 0,6

4 0,05

5 0,15

3.

= 0
0,8

=1

0,6

0,4

0,2

x 4 2 0 2 4

Esboa no teu caderno e usando as mesmas escalas, uma outra distribuio normal com um desvio padro inferior e com uma mdia superior.

Exerccios globais de 2. oportunidade

21

4.

A distribuio de probabilidade de uma dada varivel aleatria X 1 a Sabendo que dessa varivel aleatria. 2 0,2 e que 3 0,2 4 b 5 0,2 6 c

, determina a mdia e o desvio padro

5.

Considera que o consumo de gua na Escola Secundria Daniel da Silva segue uma distribuio normal em que o valor mdio 400 litros e o desvio padro de 30 litros. Usando uma calculadora determina a probabilidade de o consumo de gua, em certo dia, 5.1 5.2 5.3 variar entre 100 e 450 litros; no ultrapassar 500 litros; ser superior a 400 litros.
Pinho train station por Feliciano Guimares, http://www.lickr.com/photos/jsome1/1183813060/

Pensa e Resolve

6.

Na estao da CP do Paraimo 16 passageiros compraram cada um o seu bilhete de comboio. 7 para Aveiro (preo do bilhete 3); 5 para Coimbra (preo do bilhete 4); e 4 para o Porto (preo do bilhete 5). Escolheu-se ao acaso um destes passageiros. Seja Y a varivel aleatria que associa a cada passageiro o preo do seu bilhete. A distribuio de probabilidade associada a esta varivel dada pela tabela: 3 4 5

Determina o valor esperado E(Y) da varivel aleatria Y.

22

Exerccios globais de 2. oportunidade

7. 8.

Uma varivel aleatria X segue uma distribuio normal, de mdia 5. Indica o valor de verdade da seguinte proposio: P(X > 3) > P(X < 6) . Justiica a tua resposta. Um dardo lanado para um alvo dividido em trs zonas: A, B e C. Se o dardo for cravado na zona A, obtemos 10 pontos. Se for cravado na zona B, 2 pontos. Se for cravado na zona C, 0 pontos. O Joo lanou 100 dardos, que se repartiram da seguinte forma: 20 dardos em A, 50 em B e 30 em C. Faz uma distribuio de frequncias e calcula a mdia dos pontos obtidos (analiticamente) e o desvio-padro (com a calculadora).
Basketball hoop por Acid Pix, http://www.lickr.com/photos/acidpix/6065174738/

9.

O Joo convidou dois amigos para jogar com ele, o lvaro e a Marisa. Combinaram que cada um lanaria 12 vezes o dardo, somados os pontos obtidos em cada lanamento, deiniriam as suas classiicaes. A Marisa foi a primeira a fazer os lanamentos e obteve 24 pontos. De seguida, o lvaro fez 18 pontos. Vai agora lanar o Joo. Ser que vai ganhar o concurso ? Num jogo de basquetebol h exatamente dois resultados possveis: vitria ou derrota (se o jogo terminar empatado no tempo regulamentar so jogados prolongamentos at desempatar o jogo). Em cada jogo a probabilidade de o Estrelas da Avenida ganhar de 40%. Se o Estrelas da Avenida disputar 4 jogos num torneio de basquetebol, qual a probabilidade de ganhar exatamente 2 jogos?

10.

Relete
11. A tabela seguinte a distribuio de probabilidade de uma varivel aleatria X: 7 p Calcula o valor esperado de X: 11.1 11.2 12. Em funo de p e de q. Em funo apenas de p. 9 q 11 p 13 q

Uma Prova de avaliao constituda apenas por questes de escolha mltipla. A prova tem 4 questes e cada questo tem 5 hipteses de resposta das quais s uma certa. Se cada resposta errada desconta 3 pontos, quanto deve valer cada resposta certa para que a pontuao esperada para um aluno, que responda ao acaso a todas as questes, seja zero?

Exerccios globais de 2. oportunidade

23

C5 CAPTULO 5 ANLISE COMBINATRIA


Teenagers playing soccer in the rain por Marlon dias, http://www.lickr.com/photos/marlondias/4019108057/

Pratica
1. 2. 3. 4. Quantas matrculas de automveis diferentes podem existir no sistema atual portugus, considerando que o alfabeto tem 26 letras? Se o alfabeto portugus tivesse 23 letras como sucedia antes do Acordo Ortogrico, quantas matrculas de automveis possveis teramos a menos do que hoje? Quantas matrculas de automveis so capicuas, ou seja, os dois primeiros algarismos so iguais aos dois ltimos mas por ordem inversa e as duas letras so iguais? Pretende-se organizar um campeonato de futebol com 7 equipas. Se cada equipa encontra cada uma das outras equipas uma s vez, quantos jogos ser preciso organizar? E se cada equipa tiver de jogar com cada uma das outras equipas tanto em sua casa como fora? De quantas maneiras podes ordenar verticalmente 5 dos teus livros, de disciplinas diferentes, numa tua estante? De quantas maneiras se podem ordenar as letras da palavra LIVRO? De quantas maneiras se podem ordenar as letras da palavra LIVRO de modo que as duas vogais se mantenham nas suas posies? Quantas fotograias diferentes pode tirar uma famlia em que todos os 6 elementos da famlia icam uns ao lado dos outros? Num computador digital, um bit um dos algarismos 0 ou 1 e uma palavra uma sequncia de bits. Determina o nmero de palavras distintas de 32 bits que possvel formar. Foram oferecidos dez bilhetes para uma pea de teatro a uma turma com doze raparigas e oito rapazes. Ficou decidido que o grupo que vai ao teatro formado por cinco rapazes e cinco raparigas. 10.1 10.2 De quantas maneiras diferentes se pode formar este grupo? O Joo aluno da turma. Qual a probabilidade de o Joo pertencer ao grupo que vai ao teatro?

5.

6. 7. 8. 9. 10.

Pensa e Resolve
11. Qual seria o modo mais eicaz de aumentar o nmero de matrculas de automveis em Portugal: acrescentar um nmero ou uma letra?

24

Exerccios globais de 2. oportunidade

12.

Um professor de Matemtica deu aos alunos uma lista de exerccios, numerados de 1 a 50, e escolheu, para um teste, dois desses exerccios ao acaso. 12.1 12.2 Qual a probabilidade um aluno que fez 3/4 dos exerccios da lista ter feito os dois exerccios escolhidos pelo professor? Qual a probabilidade um aluno que fez 1/4 dos exerccios da lista ter feito um dos dois exerccios escolhidos pelo professor?

13. 14. 15. 16. 17. 18.

De quantas maneiras se podem ordenar as letras da palavra BIBLIOTECA? De quantas maneiras se podem ordenar as letras da palavra BIBLIOTECA de modo que se mantenham a primeira e a ltima letra nas suas posies? De quantas maneiras se podem ordenar as letras da palavra PACIFICA? De quantas maneiras se podem ordenar as letras da palavra PACIFICA de modo que as consoantes se mantenham nas suas posies? Quantas fotograias diferentes pode tirar uma famlia em que um elemento da famlia vai tirando a foto aos outros 5 elementos da famlia, icando sempre uns ao lado dos outros? Num grupo de cinco amigas, s uma est habilitada para conduzir. De quantas formas se podem sentar num automvel de 5 lugares, para fazer uma viagem?

Relete
19. O jogo das sete famlias constitudo por 42 cartas. Neste jogo h 7 conjuntos de cartas constitudos pelo av, av, pai, me, ilho e ilha; cada conjunto constitui uma famlia. Tiram-se do baralho de cartas, simultaneamente, 4 cartas. Determina o nmero da casos em que: 19.1 19.2 19.3 19.4 19.5 20. As 4 cartas tiradas so da mesma famlia. Entre as 4 cartas no h nenhuma carta de uma famlia dada. Entre as 4 cartas h uma carta av de uma famlia dada. Entre as 4 cartas h uma e uma s carta de uma famlia dada. Entre as 4 cartas haja apenas uma carta pai.

Uma determinada marca de CDs garante que a probabilidade de um deles estar estragado de 0,001%. Um cliente compra 50 CDs. Determina a probabilidade de: 20.1 20.2 20.3 Um deles estar estragado. No mximo um deles estar estragado. Pelo menos dois deles estarem estragados.

Exerccios globais de 2. oportunidade

25

21. 22.

Quantas retas podem ser traadas usando as letras assinaladas no cubo da igura ao lado? Qual a probabilidade de, escolhidos 3 pontos ao acaso no cubo da igura ao lado, eles deinirem um plano? Seja dada uma populao de n elementos. Indica qual o nmero de amostras ordenadas distintas, de dimenso r, que se podem selecionar desses n elementos se: 23.1 23.2 A seleo for feita com reposio. A seleo for feita sem reposio.
G B A F C

D J

23.

24.

Qual a probabilidade p de que, num conjunto de r pessoas, no haja duas a fazer anos no mesmo dia? Considera os pontos A, B, C e D representados no cubo da igura ao lado. Determina a probabilidade de, escolhidos 3 pontos ao acaso, eles deinirem um plano. Considera os pontos A, B, C, D, E e F representados no cubo da igura ao lado. Determina a probabilidade de, escolhidos 3 pontos ao acaso, eles deinirem um plano. Considera os pontos A, B, C, D, E e F representados no cubo da igura ao lado. Determina a probabilidade de, escolhidos 2 pontos ao acaso, eles deinirem uma reta.

25.

C B A

26.

27.

26

Exerccios globais de 2. oportunidade

C6 CAPTULO 6 TRINGULO
Pratica
1. Considera a seguinte parte inicial do tringulo de Pascal:

DE

PASCAL

BINMIO

DE

NEWTON

Acrescenta-lhe as duas linhas seguintes. 2. Determina os nmeros em falta no tringulo de Pascal seguinte: 1 1 1 1 1 1 1 1 1 1 1 3. 10 9 45 7 6 21 28 36 84 56 5 15 35 70 126 4 3 6 2 3 4 10 5 15 35 21 6 7 28 84 8 1 1 1 1 1 1 1 1 9 10 1 1

? ?

? ?

? ? ?

120 210 252 210

Recorrendo frmula do binmio de Newton calcula: 3.1 3.2

4.

Determina o termo em

no desenvolvimento de

Pensa e Resolve
5. a b c d e f g representa uma linha completa do Tringulo de Pascal, onde todos os elementos esto substitudos por letras. Determina essas letras.

Exerccios globais de 2. oportunidade

27

Blaise Pascal (1623-1662)

6. 7.

Determina o valor de n que veriica a seguinte condio

Determina os valores dos coeicientes numricos dos termos do 7. e 8. grau no desenvolvimento de . . .

8.

Reduz a uma forma mais simples a equao

9.

Determina o termo independente de x no desenvolvimento de

Relete
10. Determina o desenvolvimento de: 10.1

10.2 11. A partir da frmula do binmio de Newton determina um valor para a soma:

12.

Mostra, por induo matemtica, que se n um nmero natural, ento

28

Exerccios globais de 2. oportunidade

Blaise Pascal por Janmad, http://fr.wikipedia.org/wiki/Fichier:Blaise_Pascal_Versailles.JPG

C7 CAPTULO 7 FUNO
Pratica
1. Esboa o grico da funo deinida na reta real por . A partir do grico desta funo esboa os gricos das seguintes funes, indicando para cada caso o domnio, contradomnio e zeros: 1.1 1.2 1.3 1.4 1.5 2. Considera as funes deinidas na reta real por: e 2.1 2.2 3. 4. Representa-as graicamente. Determina, com aproximao at s centsimas, o conjunto soluo de . Supondo que .

EXPONENCIAL

Considera a funo f deinida por de a. Resolve as equaes: 4.1 4.2

, determina o valor exato

4.3 5. Escreve cada uma das expresses sob a forma de um produto: 5.1 5.2 5.3
Exerccios globais de 2. oportunidade

29

Pensa e Resolve
6.
Saturday Morning Caf (Cappuccino) por Frank Weber, http://www.lickr.com/photos/frawemedia/4863864661

Quando nos entregam uma bica, o caf vem muito quente e quem no pe acar precisa de esperar algum tempo para o beber. A evoluo da temperatura T (em C) em funo do tempo t (em minutos) deinida pela expresso 6.1 6.2 6.3 6.4 6.5 6.6 . Representa graicamente a funo T. A que temperatura nos entregue o caf? Quem gosta de o beber a 60 quanto tempo tem de esperar? O arrefecimento do caf mais acentuado nos primeiros dois minutos ou nos dois minutos seguintes? Em que instante que o arrefecimento mais acentuado? Que acontece se deixarmos o caf arrefecer muito tempo? Relaciona a concluso a que chegaste com a expresso de T.
(adaptado da brochura de Funes, 12. ano, ME, 1999)

7. 8.

Recorrendo calculadora resolve a equao Calcula os limites seguintes: 8.1

8.2

8.3 9. Resolve as seguintes equaes: 9.1 9.2 9.3

30

Exerccios globais de 2. oportunidade

Relete
10. H pessoas que por razes de natureza fsica ou psquica tm diiculdade em adormecer. Os mdicos dispem duma vasta gama de medicamentos que podem receitar nestes casos. Uma propriedade importante que se requer a estes medicamentos que o seu efeito desaparea antes da manh seguinte de forma que quem o toma possa retomar a sua atividade normal sem estar sonolento. Imagina que o mdico receitou a uma tua amiga um destes medicamentos. Depois de tomar algumas pastilhas, o medicamento atingiu um nvel de 4 mg/L no sangue. Com que rapidez desaparecer o efeito do medicamento? Para estudares a situao considera os dados da tabela, referentes a 4 medicamentos: Nome Triazolam Frmula

Nitrazepam

Pentobombitone Methohexitone A - dose inicial (mg/L); y - quantidade de medicamento no sangue (mg/L) x - tempo em horas desde que o medicamento chegou ao sangue. 10.1 10.2 10.3 10.4 10.5 10.6 10.7 10.8 Qual a quantidade de Triazolam no sangue ao im de 3 horas? E ao im de 10 horas? Regista numa tabela a quantidade de Triazolam nas primeiras 10 horas. Desenha um grico que possa descrever o comportamento do Triazolam. S trs destes medicamentos podero ser reais. Qual deles no ? O que aconteceria se por engano tomasses esse produto? Faz os gricos que te permitem analisar como evolui uma dose que provocou a concentrao de 4 mg/L de cada um dos medicamentos. Qual dos medicamentos te parece prefervel? Porqu? Analisa agora com algum pormenor o efeito do Triazolam. Ao im de quanto tempo se reduz a metade a quantidade de medicamento no sangue? A reduo para metade depende do tamanho da dose inicial? Como? Qual ser o efeito de tomar, hora a hora, uma dose de 4mg de Pentobombitone? Faz uma representao grica que descreva as tuas concluses.
(adaptado da brochura de Funes, 12. ano, ME, 1999)

Exerccios globais de 2. oportunidade

31

11.

O Pblico noticiou em 1995 a descoberta de uma necrpole*, na Granja dos Serres - Sintra, e o achado de seis sepulturas cujas datas, ainda desconhecidas, se podem situar desde o sc. I A.C. at ao sc. VII D.C. (* Uma necrpole um lugar onde existe uma ou mais sepulturas de tempos antigos.)
A datao da necrpole s ser esclarecida com anlises aos ossos por carbono 14 - mtodo de datao a partir de um istopo radioactivo de carbono que torna possvel determinar a idade dos materiais em anlise, uma vez que o seu tempo de desintegraao conhecido (...) jornal PBLICO, de 8 de Outubro de 1995

Tal como este artigo tambm refere, uma tcnica utilizada para descobrir a antiguidade de um achado histrico consiste na anlise de um objecto (osso, madeira, ...), medindo a quantidade do elemento radioativo carbono 14 que contm. Quando vivos, os animais e plantas tm uma quantidade constante de carbono 14, que vai diminuindo com o tempo, aps a morte, por efeito da desintegrao radioativa. Por quantidade de carbono 14 entende-se a velocidade de desintegrao de tomos de carbono 14 medida em desintegraes por minuto por grama de carbono (dmg). A quantidade q(t) de carbono 14 encontrada num objecto dada pela frmula 11.1 11.2 , em que t representa o tempo em milhares de anos.

Admitindo que os corpos encontrados nos tmulos so do sc. I a.C., que quantidade de carbono 14 deveria ser encontrada em 1995? Se o Instituto Nacional de Engenharia e Tecnologia Industrial tivesse divulgado que a quantidade de carbono 14 encontrada era de 11,3 dmg, qual seria a idade das sepulturas? Imagina que s um investigador do INETI e te pediram um artigo em que fundamentes teoricamente os resultados que divulgaste. Escreve o artigo, com o mximo de 3 pginas A4.
(adaptado da brochura de Funes, 12. ano, ME, 1999)

11.3

12.

Na cidade mongol de Ulam Bator (a capital e a maior cidade da Monglia) surgiu uma epidemia de gripe asitica. A evoluo da doena foi dada pela frmula representa a percentagem de pessoas doentes e t o tempo em dias. 12.1 12.2 12.3 12.4 onde P

Qual era a percentagem da populao doente quando se comeou o estudo da epidemia? Quando foi o pior momento da epidemia? Qual era a percentagem de doentes? A epidemia considera-se erradicada quando a percentagem de doentes for inferior a 1%. Quando aconteceu isso? No 15. dia, qual a probabilidade do presidente da cmara estar doente?
(adaptado da brochura de Funes, 12. ano, ME, 1999)

32

Exerccios globais de 2. oportunidade

C8 CAPTULO 8 FUNO
Pratica
1. Simpliica o mais possvel: 1.1 1.2 1.3 2. log 2 223 log 2 323 log 2 3 0

LOGARTMICA

Sabendo que log 7 = 0,85 calcula: 2.1 2.2 log 7 5 log 7005

3.

As calculadoras cienticas e gricas s tm nas suas teclas o logaritmo natural ou o logaritmo decimal. Para calcular logaritmos noutras bases preciso usar a frmula de mudana de base. Usando essa frmula e uma calculadora calcula: 3.1 3.2 log 3 47 log 23 274

4.

Resolve as equaes logartmicas seguintes: 4.1 4.2 log x + log 40 = 2 log 5 7 = + log(2x + 1) Calcula c(6) e c(60). Quantas unidades se produziram com um custo total de 1010 euros? 33

5.

O custo total do fabrico de x unidades dum produto , em euros c(x ) = 2x ln x + 200 . 5.1 5.2

Exerccios globais de 2. oportunidade

Pensa e Resolve
6. 7. Considera a funo g deinida por g (x ) = 3x . Determina a abcissa do grico de g cuja ordenada igual a 2. Considera que a funo f a funo logaritmo natural. Determina o mdulo da diferena entre as abcissas dos pontos do grico de f cujas ordenadas so 1 e 1. Considera as funes f e g deinidas, respectivamente por f (x ) = log 2 x e g(x ) = log 5 (x 2 + x ) Determina, recorrendo calculadora quando necessrio: 8.1 8.2 o domnio de cada uma das funes. os pontos do grico de g que esto por baixo dos do grico de f.

8.

9.

Considera que a quantidade Q(t) de uma substncia radioativa se desintegra de acordo com a frmula Q (t ) = Q e kt , onde t est expresso em minutos. Suponhamos que a meia vida, isto 0 o tempo que a substncia leva a icar reduzida a metade, de 11 minutos. Mostra que, nestas condies, k = ln 2 11 .

10.

Simpliica as seguintes expresses: 10.1 10.2 log 2(x 10 2y z 3 ) log 2 x 3 105 y

10.3 11.

ln(x + y ) ln(x 1 + y 1 )

Supe que x = log p e que y = log q . Escreve as expresses seguintes em termos de x e y: 11.1 11.2 11.3 log(p 4 3 q ) log pq p q4

34

Exerccios globais de 2. oportunidade

12.

Os logaritmos so teis para medir quantidades que variam entre valores muito pequenos e valores muito grandes. Tal o caso da acidez (pH) de um lquido, estudada na Qumica. A acidez depende da concentrao dos ies de hidrognio no lquido (expressa em moles por litro), que se designa por [H+]. O pH deinido pela expresso 12.1 A concentrao de ies de hidrognio na gua do mar de . .

Faz uma estimativa, sem usar calculadora, do pH da gua do mar. Usando uma calculadora calcula um valor aproximado do pH da gua do mar. 12.2 13. Uma soluo de vinagre tem pH igual a 3. Determina a concentrao de ies de hidrognio nessa soluo.

Determina os domnios das funes deinidas pelas expresses seguintes: 13.1 13.2 13.3 ln(1 x + 1) ln x log 2 x +3 x 4

Exerccios globais de 2. oportunidade

35

Sailing across Mediterranean por Mircea, http://www.lickr.com/photos/60265885@N03/7753745150/

Relete
14. verdade que , para todo o x real positivo? Sim ou No? Imagina que

algum no tem a tua opinio. Elabora um texto com argumentao de modo a convenc-lo. 15. Para cada uma das seguintes igualdades, indica se verdadeira para todos os valores de a e b reais positivos ou se no . Justiica devidamente cada airmao: 15.1

15.2

15.3

15.4 16. Seja x um inteiro natural positivo e seja n o nmero de algarismos da escrita decimal de x. 16.1 16.2 17. Justiica que . .

Deduz da alnea anterior qual o nmero de casas decimais de um nmero como

Resolve as seguintes inequaes: 17.1 17.2 17.3 17.4

36

Exerccios globais de 2. oportunidade

2.

Recomendaes do GAVE

No Relatrio de setembro de 2010 publicado pelo GAVE com o ttulo Um olhar sobre os resultados dos exames nacionais podem-se encontrar informaes muito interessantes sobre os aspetos em que os alunos revelam melhor e pior desempenho nos exames nacionais, assim como recomendaes para a lecionao feitas a partir dessa anlise. Documentos como estes so muito teis para os alunos e os professores, embora em cada ano os alunos e as turmas possam exibir caractersticas muito variadas. Mesmo assim, as dificuldades mais comuns so reveladas por tais documentos. Entre os aspetos onde os alunos do ensino secundrio tm melhor desempenho na disciplina de Matemtica, segundo este relatrio, esto os seguintes: No ensino secundrio, os itens com melhor desempenho, independentemente da tipologia, convocam quase sempre operaes mentais como transferir e, mais esporadicamente, argumentar, relacionar, interpretar. Os alunos tambm revelam facilidade nos itens de clculo direto ou que apelem leitura e seleo de informao. Entre os aspetos que os alunos do ensino secundrio revelam mais dificuldades encontram-se: No ensino secundrio, as maiores diiculdades prendemse com a resposta aos itens que mobilizam operaes mentais como argumentar/justificar, analisar, relacionar, em geral, e, muito pontualmente, transferir e classiicar. Tambm fraco o desempenho nos itens em que se solicita a concretizao de raciocnio dedutivo e a interpretao em contexto. O GAVE conclui ainda que, tanto no Ensino Bsico como no Ensino Secundrio os alunos revelam algumas dificuldades comuns: os examinandos revelam fragilidades no domnio da compreenso da lngua, na comunicao escrita, no recurso ao clculo, na interpretao de novas situaes e diiculdades em utilizar as capacidades grficas da calculadora. Em funo destas concluses, o relatrio do GAVE recomenda No ensino secundrio, considerase muito importante a lecionao dos problemas a partir de contextos reais e com a execuo de clculos mais complexos. Na concluso deste relatrio afirmado que O documento que agora se conclui pretende, atravs da identiicao de nveis de desempenho dos alunos, em sede de avaliao externa, contribuir para uma melhoria sustentada dos resultados, em consequncia de um progressivo upgrade da qualidade dos saberes, das competncias e do saberfazer dos nossos alunos. Nesta ordem de ideias foram selecionados para esta segunda parte algumas tarefas que permitem desenvolver as capacidades identificadas neste relatrio do GAVE como sendo as que colocam mais dificuldades aos estudantes. As tarefas so de ndole muito variada, podendo ser itens de exames ou tarefas para a sala de aula, para trabalho em pequenos grupos ou para trabalho de auto-estudo.
Recomendaes do GAVE

37

Assim, a segunda parte deste Livro de exerccios ter os seguintes captulos (o captulo 3 aparece apenas no segundo volume): Captulo 1 Resoluo de problemas da vida real Captulo 2 Problemas que envolvem clculos mais elaborados no conjunto dos nmeros reais Captulo 3 Problemas que envolvem clculos mais elaborados no conjunto dos nmeros complexos Captulo 4 Exerccios que pressupem raciocnios demonstrativos Captulo 5 Utilizar a calculadora grfica para resolver problemas

38

Recomendaes do GAVE

C1 CAPTULO 1 - RESOLUO
DE PROBLEMAS DA VIDA REAL

TR
TAREFAS
1.
RESOLVIDAS
birthday cake 08 por normanack, http://www.lickr.com/photos/29278394@N00/2789584920

O Problema dos aniversrios (1. parte) Suponhamos que estamos numa sala com 20 pessoas. Qual a probabilidade de no haver duas pessoas a fazer anos no mesmo dia?

RESOLUO
Para resolver este problema temos de partir do princpio que o ano tem 365 dias e que a taxa de nascimentos constante ao longo do ano, de modo a poder admitir que qualquer dia do ano igualmente provvel para ser o aniversrio de uma pessoa. O que pretendemos ento calcular a probabilidade de no haver repeties numa amostra de dimenso n obtida por amostragem com reposio de uma populao de dimenso N. Assim no nosso caso n = 20 e N = 365 e o nmero de casos favorveis ao acontecimento desejado dado por nmero de casos possveis Laplace, igual a
365 365

eo

. A probabilidade pedida ento, utilizando a regra de

A20

365

A '20

A20

36520

= 0,589

Note-se que este problema tem uma soluo bastante simples se se raciocinar em termos de probabilidades condicionadas. Com efeito, a 1. pessoa pode fazer anos em qualquer dia e a

Recomendaes do GAVE

39

probabilidade

365 365

. Dado que a 1. pessoa faz anos num determinado dia, a 2. pessoa

tem probabilidade

de fazer anos num dia qualquer que no o da 1. pessoa. Continuan365 do at terminar a 20. pessoa, temos que a probabilidade pretendida o produto das probabilidades calculadas.
Stack of envelopes por stackorama, http://www.lickr.com/photos/slackorama/326182675

364

A probabilidade de numa sala com 20 pessoas haver pelo menos duas pessoas a fazer anos no mesmo dia portanto 1 0,589 = 0,411.
(adaptado da brochura Pro ba bilid a d es 1 2 , ME, 1 9 9 9 )

2.

Cartas e envelopes Uma secretria muito desarrumada tinha 3 cartas para meter em 3 envelopes, mas caiu tudo ao cho e ela meteu as cartas nos envelopes sem tomar ateno aos nomes. Uma das cartas era para o Senhor Silva. 2.1 2.2 Qual a probabilidade de ele receber a carta que lhe era dirigida? Qual a probabilidade de pelo menos uma pessoa receber a carta que lhe era destinada?

RESOLUO
2.1 Para resolver esta questo preciso admitir que se as cartas foram colocadas aleatoriamente nos envelopes, ento a carta para o Senhor Silva tem igual probabilidade de aparecer num qualquer dos envelopes. Assim a probabilidade de a secretria meter a carta no envelope certo precisamente .

2.2 Para sabermos se pelo menos uma pessoa recebeu a carta que lhe era destinada, temos de considerar os casos em que uma pessoa recebeu a carta que lhe era destinada e os casos em que duas pessoas receberam a carta que lhes era destinada e os casos em que as trs pessoas receberam a carta que lhes era destinada. Teremos de ter cuidado em subtrair os 40
Recomendaes do GAVE

casos em que se verificam simultaneamente duas dessas situaes atendendo propriedade 5 do Manual (volume 1, captulo 2) P (A B ) = P (A) + P (B ) P (A B ) e que generalizado na tarefa 45 deste volume. Designemos as cartas por C1, C2 e C3 e os destinatrios corretos destas cartas por S1, S2 e S3. i) casos em que uma pessoa recebeu a carta que lhe era destinada: Considerando por exemplo a carta C1, os casos em que vai parar a S1 so 2! (permutaes dos destinatrios C2 e C3). Os casos possveis so permutaes de 3 destinatrios, ou seja 3!. Logo a probabilidade neste caso 2! 3! Como para a carta C2 e a carta C3 a situao idntica, a soma das probabilidades de uma pessoa receber a carta que lhe era destinada dada por 3 2! 3! =1

ii) casos em que que duas pessoas receberam a carta que lhes era destinada: Considerando por exemplo as cartas C1 e C2, os casos em que vo parar a S1 e S2 so as possibilidades que sobram para a terceira carta que s uma. Os casos possveis so novamente permutaes de 3 destinatrios, ou seja 3! Logo a probabilidade neste caso 1 3! Temos 3 2 possibilidades para tomarmos duas das cartas de cada vez. Logo a soma das probabilidades de duas pessoas receberem a carta que lhes era destinada dada por 3 1 3! 2 iii) casos em que que as trs pessoas receberam a carta que lhes era destinada: H apenas uma possibilidade de as trs cartas chegarem ao seu destinatrio correto que a de C1, C2 e C3 chegarem exatamente a S1, S2 e S3 respetivamente. A probabilidade de isso
Recomendaes do GAVE

41

acontecer ento 1 3! iv) concluso: A probabilidade pedida ser ento a soma das probabilidades de uma pessoa receber a carta que lhe era destinada a que temos de subtrair a soma das probabilidades de duas pessoas receberam a carta que lhes era destinada pois estes casos j foram necessariamente contabilizados antes a que temos de adicionar os casos em que as trs pessoas receberam a carta que lhes era destinada pois estes foram subtrados uma vez a mais. Assim a probabilidade pedida igual a 1 1 1 1 1 1 3 + =1 + = 2 3! 3! 2 6 3
(adaptado da brochura Pro ba bilid a d es 1 2 , ME, 1 9 9 9 )

3.

A raspadinha Numa raspadinha esto em jogo 100 bilhetes, repartidos da seguinte maneira: uma raspadinha tem um prmio de 100 euros, nove raspadinhas tm um prmio de 10 euros e nenhuma outra raspadinha tem prmio. Cada raspadinha custa 3 euros e os prmios esto distribudos ao acaso nas raspadinhas. Seja X a varivel aleatria que mede o ganho de cada jogador (diferena entre o que ganha no prmio e o que gastou a comprar a raspadinha). 3.1 3.2 Determina a distribuio de probabilidades da varivel aleatria X. O jogo justo para os jogadores ou favorece os organizadores da raspadinha? Justiica a resposta.

RESOLUO
3.1 A varivel aleatria X s toma trs valores diferentes: 97 se o jogador ganhar o prmio de 100 euros, 7 se o jogador ganhar o prmio de 10 euros e 3 se o jogador no ganhar qualquer prmio. Como os prmios esto distribudos ao acaso pelas raspadinhas as probabilidades respetivas so as seguintes: 97 7 3

42

Recomendaes do GAVE

3.2 Para determinar se o jogo justo ou no temos de calcular o valor esperado ou valor mdio da varivel aleatria X s. Temos

Podemos assim concluir que o jogo favorece os organizadores visto que o ganho esperado de um jogador negativo. Ou seja, se o jogador jogar muitas vezes ganhar em mdia 1,1 euros, ou seja, perder dinheiro. 4. Baile de Finalistas Numa turma do 12. ano da Escola Secundria Lus de Albuquerque, a distribuio dos alunos por idade e sexo a seguinte: 12. X rapazes raparigas 16 anos 6 5 17 anos 8 7

Para formar uma comisso que vai preparar um baile de finalistas, vo ser sorteadas trs rapazes e duas raparigas desta turma. 4.1 4.2 Qual a probabilidade de a comisso icar constituda apenas por jovens de 16 anos? Apresenta o resultado na forma de dzima, com quatro casas decimais. Admite agora que j esto sorteados quatro dos cinco jovens que vo constituir a comisso: os trs rapazes e uma rapariga, a qual tem 16 anos de idade. Para a comisso ficar completa, falta, portanto, escolher aleatoriamente uma rapariga. Seja X a varivel aleatria: nmero de raparigas de 17 anos que a comisso vai incluir. Constri a tabela de distribuio de probabilidades da varivel X. Apresenta as probabilidades na forma de frao.
romeojuliet-spr12-7 por KCBalletMedia, http://www.lickr.com/photos/67555847@N06/6893010197/

Recomendaes do GAVE

43

RESOLUO
4.1 A comisso constituda por 3 rapazes e 2 raparigas. Ora, temos 12 raparigas. primeira vista poder parecer-nos que existem 12 11 = 132 maneiras diferentes de escolher, ao acaso, duas dessas 12 raparigas. Mas, essa suposio est errada. Admitamos que queremos escolher duas raparigas de entre as seguintes trs: {Ana, Beatriz, Celina}. fcil concluir que existem apenas trs possibilidades: {Ana, Beatriz}, {Ana, Celina} e {Beatriz, Celina}. No seis: (Ana, Beatriz), (Beatriz, Ana), (Ana, Celina), (Celina, Ana), (Beatriz, Celina) e (Celina, Beatriz). Isto , como no interessa a ordem dos dois elementos considerados, o valor procurado , que traduz o nmero de subconjuntos de dois elementos que se podem obter de um conjunto de trs elementos. Admitamos agora que pretendemos escolher trs rapazes de entre quatro: {Abel, Belmiro, Carlos, Daniel}. imediato concluir que existem apenas maneiras, no

4 3 2 = 24: {Abel, Belmiro, Carlos}, {Abel, Belmiro, Daniel}, {Abel, Carlos, Daniel} e {Belmiro, Carlos, Daniel}. Porque que divide por 3 2? Basta reparar que cada um desses subconjuntos de trs elementos d origem a 3 2 = 6 ternos ordenados com esses trs elementos. Portanto, regressando ao problema, conclumos existirem lecionar duas das doze raparigas e rapazes. maneiras de se-

maneiras de selecionar trs dos catorze

Logo, o nmero de casos possveis 364 66 = 24024. De forma anloga, conclui-se que o nmero de casos favorveis isto , o nmero de maneiras de escolher 3 rapazes de 16 anos, de entre 6, e de escolher 2 raparigas de 16 anos, de entre 5. Logo, a probabilidade pedida

4.2 Para terminar a constituio da comisso falta apenas escolher uma rapariga, de entre 11 disponveis: 4 delas com 16 anos e 7 delas com 17 anos. Portanto, a varivel aleatria X pode assumir os valores: 0 e 1.

44

Recomendaes do GAVE

Assim: P(X = 0) = P(escolher uma rapariga de 16 anos) =

P(X = 0) = P(escolher uma rapariga de 17 anos) = Logo, a tabela de distribuio de probabilidades da varivel X : 0 4 11 5. Trs Bilhetes de Cinema Resolve por quatro processos o seguinte problema: A professora de Histria resolveu levar os seus 15 alunos a ver um filme. Como o cinema tem filas de precisamente 15 cadeiras, comprou uma fila inteira e distribuiu os bilhetes ao acaso pelos alunos. A Ana, a Bela e a Carla so muito amigas e gostavam de ficar as trs juntas e numa das pontas da fila. Qual a probabilidade de isso acontecer? 1 7 11

RESOLUO
1. Processo Vamos pensar apenas nos trs bilhetes destinados s trs amigas, no nos interessando a ordem como elas ocuparo depois esses trs lugares. O espao de resultados o conjunto dos ternos no ordenados. Por exemplo, um dos seus elementos o terno {5,7,15}, que corresponde s trs amigas receberem os bilhetes 5, 7 e 15 embora no saibamos o lugar exato em que cada uma delas se vai sentar. Os casos possveis so as diferentes maneiras de elas receberem os 3 bilhetes de um conjunto de 15, ou seja, todos os ternos no ordenados formados a partir do conjunto de 15 bilhetes. Casos possveis: Casos favorveis: apenas 2, ou recebem os bilhetes 123 ou os bilhetes 131415. Logo a probabilidade pedida 2 455 .

Recomendaes do GAVE

45

2. Processo Vamos pensar nos trs bilhetes destinados s trs amigas, mas interessando-nos agora a ordem como elas ocuparo depois esses trs lugares. Continuamos a ignorar os outros 12 bilhetes. O espao de resultados o conjunto dos ternos ordenados. Por exemplo, um dos seus elementos o terno , ou seja, a Ana fica no lugar 5, a Bela no 7 e a Carla no 15. Os casos possveis so portanto as diferentes maneiras de elas receberem 3 bilhetes de um conjunto de 15, mas em que a ordem por que recebem os bilhetes importante. Casos possveis:
15

A3 = 2730

Casos favorveis: Se os bilhetes que elas receberem forem 1, 2 e 3, como a ordem interessa, h seis maneiras de elas os ocuparem (so as permutaes de 3). O mesmo se passa para os bilhetes 13, 14 e 15. Logo, os casos favorveis so 2 P3 = 12 . Logo a probabilidade pedida 3. Processo Desta vez vamos considerar todas as maneiras como os 15 alunos se podem sentar nos 15 lugares. O espao de resultados constitudo por todas as permutaes dos 15 alunos pelas cadeiras. Os casos possveis so portanto as permutaes de 15. Casos possveis: P = 15! 15 Casos favorveis: Se as trs amigas ficarem nos lugares 1, 2 e 3, podem permutar entre si, e os outros 12 alunos tambm. O mesmo se passa se ficarem nos trs ltimos lugares. Ento os casos favorveis so 2 P3 P12 . Logo a probabilidade pedida 4. Processo Vamos calcular a probabilidade pedida admitindo que os bilhetes vo ser entregues um a um s trs amigas. A primeira vai receber o seu bilhete. Dos 15 lugares, h 6 que lhe servem (os trs primeiros e os trs ltimos). Chegou a vez da segunda. H 14 bilhetes e a ela s servem os dois lugares que restam na ponta onde a primeira ficou. 46
Recomendaes do GAVE

12 2730

2 455

2 3! 12! 15!

12 15 14 13

2 455

Finalmente, a terceira, dos 13 bilhetes restantes, tem de receber o nico que sobra na ponta onde esto as amigas. Logo a probabilidade pedida 621 15 14 13 = 6 15 2 14 1 13 = 2 455 .

NO ESQUECER

Uma questo que se coloca muitas vezes perante os problemas de Probabilidades o facto de existirem vrios processos de os resolver. Normalmente isso sucede por, perante a situao descrita no problema, se poderem considerar diferentes espaos de resultados conforme a abordagem que se faa. Para calcular a probabilidade aplicando a regra de Laplace, devemos dividir o nmero de casos favorveis pelo nmero de casos possveis. Ora, a cada espao de resultados ir corresponder um diferente nmero de casos possveis e, claro, um diferente nmero de casos favorveis. O principal cuidado a ter usar exatamente o mesmo mtodo na contagem dos casos favorveis e na contagem dos casos possveis, ou seja, no mudar de espao de resultados a meio da resoluo.
(adaptado de Jos Paulo Viana, Escola Secundria Verglio Ferreira, Lisboa)

TP
TAREFAS PROPOSTAS
6. O TOTOLOTO 6/49 O Totoloto surgiu em 1985. Criado pelo Decreto-Lei n. 382/82 de 15 de Setembro s mais tarde, atravs do Decreto-Lei n. 84/85, de 28 de Maro, o Estado concedeu SCML o direito sua organizao e explorao. O primeiro concurso realizou-se a 30 de Maro desse ano. O jogo consiste na escolha de seis nmeros, entre 49 possibilidades. Assim, os prognsticos so efectuados traando as cruzes nos quadradinhos e estabelecendo conjuntos de seis nmeros. Os prmios so atribudos a partir do acerto em trs dos nmeros escolhidos. As apostas simples tm de ser em nmero par (2, 4, 6, 8 e 10 apostas), comeando pelos dois primeiros conjuntos da esquerda e continuando sem intervalo. Em cada conjunto, marcam-se com cruzes (X), os seis nmeros escolhidos. As apostas mltiplas fazem-se sempre no conjunto 1 dos bilhetes. Podem ser preenchidos 7 a 12 nmeros, assinalando o quadradinho correspondente. No incio de 1988 surgiu uma nova modalidade de aposta mltipla, o 5/44. O apostador escolhe 5 nmeros fixos que combinam uma vez, com cada um dos restantes. O bilhete de cinco semanas permite participar em cinco concursos seguidos, com os mesmos conjuntos de nmeros.

Recomendaes do GAVE

47

6.1 6.2 6.3 7.

A quantas apostas simples corresponde a aposta mltipla de 11 cruzes? A quantas apostas simples corresponde a aposta mltipla de 5/44? Supe que izeste uma aposta mltipla, assinalaste 12 cruzes e acertaste em 3 delas. Quantos quintos prmios (aposta com 3 nmeros certos) ganhaste?

H N pessoas e cada uma pe o respectivo chapu numa caixa. Qual a probabilidade de uma determinada pessoa retirar o prprio chapu? Qual a probabilidade de que pelo menos uma pessoa escolha o chapu correto?
(adaptado da brochura Probabilidades 12, ME, 1999)

8.

Numa raspadinha esto em jogo 200 bilhetes, repartidos da seguinte maneira: duas raspadinhas tm um prmio de 200 euros, 18 raspadinhas tm um prmio de 20 euros e nenhuma outra raspadinha tem prmio. Cada raspadinha custa 3 euros e os prmios esto distribudos ao acaso nas raspadinhas. Seja X a varivel aleatria que mede o ganho de cada jogador (diferena entre o que ganha no prmio e o que gastou a comprar a raspadinha). 8.1 8.2 Determina a distribuio de probabilidades da varivel aleatria X. Sem efetuares qualquer clculo e olhando para a tarefa 3, parece-te que este jogo justo para os jogadores ou favorece os organizadores da raspadinha? Efetua os clculos e conclui. Que alteraes podes efetuar nas regas da raspadinha de modo que o jogo nem favorea os jogadores nem os organizadores?

8.3

9.

Um concurso televisivo utiliza um dispositivo chamado aparelho ou caixa de Galton, para determinar os prmios que os concorrentes ganham. Um disco largado do topo do aparelho e vai batendo sucessivamente nos pinos do aparelho at atingir as posies A, B, C, D, E ou F.

48

Recomendaes do GAVE

9.1 9.2 9.3

Quantos caminhos existem para o disco chegar posio A? E posio B? Mostra que o nmero de caminhos que h at chegar a cada pino exatamente igual aos nmeros em posio semelhante do tringulo de Pascal:
1 1 1 1 1 1 A 5 B 4 10 C 3 6 10 D 2 3 4 5 E 1 1 1 1 1 F

(adaptado da brochura Probabilidades 12, ME, 1999)

10.

O Nuno inventou o seguinte jogo de apostas, para se entreter com os seus colegas do 12. ano: cada aposta consiste em marcar n nmeros de um total formado pela lista: 1, 2, 3, 4, 5, 6, 7, 8, 9, 10, 11, 12, e quer saber quanto deve valer n para assegurar que cada um dos 120 alunos possa fazer uma aposta distinta. Um grupo de 10 amigos quer fazer um campeonato de Poker, pelo que decidem organizar partidas (de quatro) de todas as formas possveis. 11.1 11.2 Quantas partidas so possveis? Se jogarem 10 partidas por semana: 11.2.1 Quanto tempo demorariam a terminar o campeonato? 11.2.2 Quantas partidas jogar cada um ?

11.

12.

Cinco pessoas, A, B, C, D e E, devem pronunciar-se num discurso. De quantas maneiras se podem ordenar as intervenes de cada um, se D no puder falar antes de A? 49

Recomendaes do GAVE

13.

Determina o nmero de rectas distintas que podem passar por oito pontos do plano, 13.1 13.2 13.3 se esto dispostos de maneira que trs quaisquer deles no esto alinhados; se quatro deles esto alinhados e os outros quatro tambm; se os oito pontos so vrtices de um quadrado e os pontos mdios dos seus lados.

14.

Considera os oito pontos que so vrtices de um cubo. 14.1 14.2 14.3 14.4 14.5 Quantas rectas distintas determinam? E quantos tringulos? Destes, quantos so rectngulos e quantos so equilteros? E quantos quadrados? E quantos rectngulos? E quantos planos?

15.

Pintam-se as quatro faces de um tetraedro regular com duas cores distintas. Quantos tetraedros diferentes podemos obter? E se pintarmos com trs cores diferentes? E se pintarmos com quatro? O Joo tem, no bolso, seis moedas: duas moedas de 1 euro e quatro de 50 cntimos. O Joo retira, simultaneamente e ao acaso, duas moedas do bolso. 16.1 Seja X a quantia, em euros, correspondente s moedas retiradas pelo Joo. Constri a tabela de distribuio de probabilidades da varivel X, apresentando as probabilidades na forma de frao irredutvel. Depois de ter retirado as duas moedas do bolso, o Joo informou a sua irm Ins de que elas eram iguais. Ela apostou, ento, que a quantia retirada era de 2 euros. Qual a probabilidade de a Ins ganhar a aposta? Apresenta o resultado sob a forma de frao irredutvel.
x

16.

16.2

17.

4 A funo P (x ) = 22500 3

deinida para x 0, usada para determinar o valor de um

carro (em euros) x anos depois da sua compra. 17.1 17.2 17.3 18. Qual o custo inicial do carro? Determina o custo de um carro um ano e meio depois da compra. Quanto desvaloriza o carro ao ano?

Um arquiteto resolveu usar a funo logartmica para fazer o arco de uma porta, como mostra a igura seguinte.

50

Recomendaes do GAVE

y 2 B

10

O arco AB parte da funo definida por y = ln x . O arco BC simtrico do arco AB relativamente recta BD. 18.1 18.2 19. Deine uma funo por ramos de modo que represente o arco AB e o arco BC. Determina a altura mxima da porta (isto , a do arco medido sobre a reta BD).

Financiamento para a viagem de inalistas Podes observar na figura da tarefa 9 o aparelho de Galton, que pode ser utilizado em concursos. Os alunos de uma turma do 12. ano da Escola Secundria de Cima pensam utilizar um aparelho anlogo, mas com 9 linhas, para promover um concurso destinado a angariar financiamento para ajudar a pagar a viagem de finalistas. Pensam pedir um pagamento de 3,5 euros por cada aposta, ou seja, por cada disco lanado. Os jogadores podero obter um dos prmios cujo valor consta no fundo do aparelho, como podes observar no esquema imediatamente abaixo: A 100 B 20 C 10 D 3 E 1 F 1 G 3 H 10 I 20 J 100

Noutra escola, a Secundria de Baixo, os alunos de outra turma do 12. ano resolveram promover outro tipo de concurso para fim anlogo ao que se destina o concurso dos seus colegas de Cima. Criaram uma espcie de Euromilhes, o Baixocentenas, a ser realizado semanalmente que, ao contrrio do euromilhes, no d lugar diviso do prmio pelos apostadores premiados. Quem acertar recebe integralmente o valor referente ao prmio. Podes observar na figura seguinte um boletim desse concurso.

Recomendaes do GAVE

51

A seguir podes observar uma tabela de distribuio de probabilidades da varivel Y: valor ganho pelo jogador numa aposta, relativa ao concurso Baixocentenas. Y = yi pi = P Y = yi 200 50
1 330

0
263 264

1 1320

Responde s seguintes questes, considerando que os custos para alm dos resultantes dos pagamentos dos prmios (aparelho, bilhetes do Baixocentenas, impostos, etc) so suportados por patrocinadores externos em troco de publicidade. 19.1 Constri uma tabela de distribuio de probabilidade relativa ao concurso a realizar na E. S. de Cima, considerando a varivel aleatria X: valor ganho pelo apostador numa jogada. Calcula o lucro ou prejuzo esperado pelo apostador em cada aposta no concurso da E. S. de Cima. Explica os valores de probabilidade que constam da tabela de distribuio de probabilidades relativa ao concurso Baixocentenas. Se criassem, no Baixocentenas, um 3. prmio para os apostadores que acertarem os nmeros mas falharem as letras (3 nmeros + 0 letras), o que seria mais provvel a um apostador: acertar no 1. prmio ou no 3. prmio? Considerando o concurso Baixocentenas tal como est previsto, com os dois prmios, calcula o lucro/prejuzo esperado pelo jogador em cada aposta. Tendo em considerao os dois concursos, elabora uma redaco em que reiras os seguintes aspectos: - opinio acerca do melhor concurso, tendo em considerao a rentabilidade por aposta; - cumprimento do objectivo a que se destinam os concursos e riscos associados, utilizando argumentos relativos a lei dos grandes nmeros e viabilidade prtica da implementao de cada concurso; - sugestes de eventuais alteraes a introduzir em cada projeto de modo a aumentar 52
Recomendaes do GAVE

19.2 19.3 19.4

19.5 19.6

o lucro esperado pelos alunos e o interesse de potenciais jogadores. Na redao sero valorizados os argumentos matemticos utilizados, cujos clculos no precisas de repetir se j estiverem nas respostas s questes anteriores (basta invoc-los), mas tambm a apresentao, o encadeamento lgico, a clareza, a correo e a criatividade. (Nota: Se este trabalho te der alguma ideia para aplicares, deves ter muita ateno ao contexto legal.)

EM
QUESTES
20.
DE ESCOLHA MLTIPLA

A Patrcia tem uma caixa com cinco bombons de igual aspeto exterior, mas s um que tem licor. A Patrcia tira, ao acaso, um bombom da caixa, come-o e, se no for o que tem licor, experimenta outro. Vai procedendo desta forma at encontrar e comer o bombom com licor. Seja X a varivel aleatria nmero de bombons sem licor que a Patrcia come. Qual a distribuio de probabilidades da varivel X?

(A)

0,2 0,2 0,2 0,2 0,2 (B) 0 1 2 3 4

0,1 0,1 0,2 0,2 0,4 (C) 1 2 3 4 5

0,2 0,2 0,2 0,2 0,2

Recomendaes do GAVE

53

(D) 1 2 3 4 5

0,1 0,1 0,2 0,2 0,4 21. Numa caixa esto trs cartes, numerados de 1 a 3. Extraem-se ao acaso, e em simultneo, dois cartes da caixa. Seja X : o maior dos nmeros sados. Qual a distribuio de probabilidades da varivel X? (A) 2 3

(B) 2 3

(C) 1 2 3

(D) 1 2 3

54

Recomendaes do GAVE

22.

Numa caixa esto bolas brancas e bolas pretas. Extraem-se ao acaso, e em simultneo, trs bolas da caixa. Seja X o nmero de bolas brancas extradas. Sabe-se que a distribuio de probabilidades da varivel aleatria X : 1 2 3 a a Qual a probabilidade de se extrarem menos de trs bolas brancas? (A)

(B)

(C)

(D) 23. O Joo vai lanar seis mil vezes um dado equilibrado, com as faces numeradas de 1 a 6, e vai adicionar os nmeros sados. De qual dos seguintes valores de esperar que a soma obtida pelo Joo esteja mais prxima? (A) 20000 (B) 21000 (C) 22000 (D) 23000

Recomendaes do GAVE

55

C2 CAPTULO 2 - PROBLEMAS
QUE ENVOLVEM CLCULOS MAIS ELABORADOS NO CONJUNTO DOS NMEROS REAIS

TR
TAREFAS
24.
RESOLVIDAS

Demonstra que o nmero

um nmero inteiro.

RESOLUO
Temos que

56

Recomendaes do GAVE

Trail por Dennnis Vu, http://www.lickr.com/photos/dennis_vu/4229923760/

Mas as combinaes de 100 elementos de 25 a 25 do o nmero de arranjos diferentes de 25 elementos sem interessar a ordenao, que se podem obter quando temos nossa disposio uma centena de elementos. Esse nmero necessariamente um nmero inteiro, logo fica provado o que pretendamos (sem ter necessidade de efetuar todos os clculos!).

25.

Consideremos que temos dois baralhos de 32 cartas. Vamos chamar-lhes baralhos 1 e 2. 25.1 Tira-se ao acaso uma carta em cada um dos baralhos 1 e 2. Consideremos os acontecimentos

A: Obter 2 cartas de s B: Obter pelo menos um s Calcula P(A) e P(B). 25.2 Misturam-se as cartas dos dois jogos e tiram-se sucessivamente e sem reposio duas cartas ao acaso. Calcula P(A) e P(B).

RESOLUO
25.1 H 32 cartas do baralho 1 e 32 do baralho 2. Como se tira ao acaso uma carta em cada um dos baralhos 1 e 2, pelo princpio bsico da Anlise Combinatria, h 32 32 = 1024 possibilidades. H 4 possibilidades de tirar um s do primeiro baralho e quatro de tirar um no segundo baralho. Ou seja, h 4 4 = 16 possibilidades de tirar 2 cartas de s. A probabilidade do acontecimento A ento

Consideremos agora o acontecimento B Obter pelo menos um s. Teremos de ver como obter um s, dois Ases, 3 Ases e 4 Ases. Neste caso ser mais fcil estudar o acontecimento contrrio de B: : No obter qualquer carta de s Como h 28 cartas que no so s no primeiro baralho e outras tantas no segundo baralho, pelo princpio bsico da Anlise Combinatria, conclumos que h 28 28 = 784 possibilidades. A probabilidade do acontecimento

Ento

Recomendaes do GAVE

57

25.2 Como se misturam as 32 cartas de cada um dos dois baralhos e se tiram sucessivamente e sem reposio duas cartas ao acaso, h 64 possibilidades diferentes para a primeira carta e 63 possibilidades para a segunda carta. O nmero de casos possveis ento 64 63 = 4032. Para o acontecimento A h 8 possibilidades de obter um s na primeira carta e 7 possibilidades de obter um s na segunda carta; ou seja, h no total 8 7 = 56 casos favorveis pelo que

Para o acontecimento B podemos mais uma vez recorrer ao acontecimento , o acontecimento contrrio de B. Temos que ento haver 56 possibilidades de no sair s na primeira carta; e ento haver 55 possibilidades de no sair s na segunda carta. Assim,

Logo

26.

Usando a frmula do binmio de Newton calcula

RESOLUO
A frmula do Binmio de Newton diz que

No caso n = 6, a = 2x e b = 4y. Podemos dizer que todas as parcelas tm a forma

( )a
n k

n k k

com k a variar de 0 a 6. No nosso caso, todas as parcelas tero a forma

58

Recomendaes do GAVE

Como k est a variar de 0 a 6 temos ento que

Que podemos fazer para termos a certeza de que no nos enganmos nos clculos? Basta observarmos que a soma dos expoentes de a = 2x e de b = 4y e portanto de x e de y sempre igual ao expoente n = 6. Concluindo, vem ento

27.

Calcula

RESOLUO
Temos que
100

C 99 =

100! 99!1!

= 100

pois 100! = 100 99! Um modo mais simples para calcular este valor relembrar a propriedade
n

C k = nC n k

para concluir que


100

C 99 =

100

C 1 = 100 59

Recomendaes do GAVE

TP
PROPOSTAS

28.

Numa corrida de cavalos, h 18 cavalos a participar. As apostas desportivas normalmente incidem em acertar nos trs primeiros lugares chegada, por ordem ou sem interessar a ordem.

28.1 28.2 29.

Qual a probabilidade de acertar nos trs primeiros classiicados, supondo que a aposta totalmente aleatria? Suponhamos que os trs cavalos a chegar em primeiro lugar so o 9, o 17 e o 12 por esta ordem. Qual a probabilidade de acertar nestes trs lugares sem interessar a ordem?

Usando a frmula do binmio de Newton calcula o 5. termo do desenvolvimento de

29.1

29.2

30. 31. 32.

Calcula, usando a frmula do binmio de Newton, Simpliica a frao .

Determina p de modo que seja ao mesmo tempo e

60

Recomendaes do GAVE

Horse racing event por Tsutomu Takasu, http://www.lickr.com/photos/gowestphoto/3921760653/

TAREFAS

33.

Calcula: 33.1 33.2 33.3 33.4

EM
QUESTES
DE ESCOLHA MLTIPLA

34. (A) (B) (C) (D) 35.

igual a:

A soma dos trs primeiros elementos de uma certa linha do Tringulo de Pascal 121. Qual o terceiro elemento da linha seguinte? (A) 78 (B) 120 (C) 91 (D) 136

36.

Uma certa linha do Tringulo de Pascal tem quinze elementos. Qual o sexto elemento dessa linha? (A) (B) (C) (D)

Recomendaes do GAVE

61

37.

O quarto nmero de uma certa linha do Tringulo de Pascal 19600. A soma dos quatro primeiros nmeros dessa linha 20876. Qual o terceiro nmero da linha seguinte? (A) 169247 (B) 175324 (C) 184756 (D) 193628

38.

Numa certa linha do Tringulo de Pascal, o segundo elemento 2009. Quantos elementos dessa linha so maiores do que um milho? (A) 2004 (B) 2005 (C) 2006 (D) 2007

39.

O penltimo nmero de uma certa linha do Tringulo de Pascal 10. Qual o terceiro nmero dessa linha? (A) 11 (B) 19 (C) 45 (D) 144

40.

A soma dos dois ltimos elementos de uma certa linha do Tringulo de Pascal 31. Qual o quinto elemento da linha anterior? (A) 23751 (B) 28416 (C) 31465 (D) 36534

62

Recomendaes do GAVE

C4 CAPTULO 4 - EXERCCIOS QUE PRESSUPEM RACIOCNIOS DEMONSTRATIVOS TR


TAREFAS
41.
RESOLVIDAS

Demonstra que 41.1 41.2

RESOLUO
41.1 Temos que temos que e so disjuntos, pelo que, pelo 3. axioma da Probabilidade,

Assim, supondo que P(B) > 0 e usando a definio de probabilidade condicionada, temos que

Logo,
c.q.d. Recomendaes do GAVE

63

41.2 Sabemos que condicionada podemos escrever

. Pela definio de probabilidade

Logo,

.
c.q.d.

42.

Mostra, por reduo ao absurdo, que se

ento

RESOLUO
Suponhamos ento que, em vez da concluso pretendida, se teria a concluso contrria, isto , que

Ento, como

so disjuntos, pelo 3. axioma viria

Mas como, por hiptese,

ento ser

pelo que vir

Como supusemos que

vir que

e isto contradiz o 1. axioma. Chegmos a um absurdo, pelo que a hiptese feita falsa e assim conclumos que .
c.q.d.

64

Recomendaes do GAVE

TP
TAREFAS
43.
PROPOSTAS

Demonstra que: P (A B C ) = P (A )P (B | A )P[C | (A B )]

44.

Relete sobre a veracidade da seguinte airmao: O nmero de diagonais de um polgono regular de n lados calcula-se pela frmula n(n 3), porque aplicando o princpio da multiplicao, de cada um dos n vrtices saem n 3 diagonais.

45.

Usando um contraexemplo mostra a falsidade da airmao:

46.

Prova que, dados os acontecimentos A, B e C, se tem P(A B C ) = P(A) + P(B ) + P(C ) P(A B ) P(A C ) P(B C ) + P(A B C )

47.

Prova, por reduo ao absurdo, que se

ento

Recomendaes do GAVE

65

C5
CAPTULO 5 - UTILIZAR
A CALCULADORA GRFICA PARA RESOLVER PROBLEMAS

TR
TAREFAS
48.
RESOLVIDAS

O Problema dos aniversrios (2. parte) Qual o nmero mnimo de pessoas que preciso ter numa sala para que a probabilidade de haver pelo menos duas a fazer anos no mesmo dia seja superior a 50%?

RESOLUO
Intuitivamente parece que tero de existir mais de 150 pessoas na sala para haver 50% de probabilidades de duas pessoas festejarem o seu aniversrio no mesmo dia. Contudo, a Matemtica vai mostrar algo de surpreendente. Para resolver este problema de modo simples temos de partir do princpio que o ano tem 365 dias e que a taxa de nascimentos constante ao longo do ano, de modo a poder admitir que qualquer dia do ano igualmente provvel para ser o aniversrio de uma pessoa. Vamos calcular as sucessivas probabilidades de no haver duas pessoas a fazer anos no mesmo dia, comeando com uma nica pessoa na sala e fazendo entrar as outras uma a uma. Pararemos logo que a probabilidade seja inferior a 0,5. Se s houver 1 pessoa na sala, ela poder fazer anos em qualquer um dos 365 dias. A probabilidade de isso acontecer P(1) = = 1.

Entra a segunda pessoa na sala, que tem de fazer anos num dia diferente da primeira. Servem 364 dos 365 dias e a probabilidade de isso acontecer coincidncia de aniversrios das duas pessoas ento P(2) = 0,9973 . A probabilidade de no

Entra a terceira pessoa na sala, que tem de fazer anos num dia diferente das duas anteriores. Servem 363 dos 365 dias e a probabilidade de isso acontecer coincidncia dos trs aniversrios ento 363 365 . A probabilidade de no

66

Recomendaes do GAVE

P(3) = Para 4 pessoas: P(4) = fcil agora fazer a generalizao para n pessoas: P(n) =

0,9918

0,9836

Agora vamos procurar o menor valor de n que faz com que P(n) seja inferior a 0,5. Convm usar a calculadora ou o computador. Colocamos em Y1 a funo P(n), em Y2 a funo 1 P(n), que a probabilidade de haver pelo menos duas pessoas a fazer anos no mesmo dia, e fazemos uma tabela para os sucessivos valores de n.

Vemos ento que bastam 23 pessoas para que a probabilidade de haver duas pessoas a festejar o aniversrio no mesmo dia seja superior a 50%. O resultado surpreendentemente baixo. Com 30 pessoas, a probabilidade j superior a 70%, e com 41 pessoas superior 90%. Com 57 chega-se aos 99% e com 70 ultrapassa-se os 99,9%.
(adaptado da brochura Probabilidades 12, ME, 1999)

49.

Gripe Asitica Numa cidade surgiu uma epidemia de gripe asitica. Determinou-se que a evoluo da doena era dada pela frmula

onde P representa a percentagem de pessoas infetadas e t o tempo em dias aps a declarao da epidemia pelo Servio Nacional de Sade (SNS). 49.1 Determina, analiticamente, o perodo de tempo (em horas) em que a percentagem de pessoas infetadas foi superior ou igual existente no momento da deteco da epidemia.

Recomendaes do GAVE

67

49.2

Quando foi declarada a epidemia, o SNS sossegou a populao da cidade informando que situao no era muito grave, pois tinham sido tomadas todas as medidas recomendadas e a epidemia seria erradicada em menos de uma semana. Numa pequena composio, comenta o teor das declaraes do SNS tendo em conta que a) A epidemia se considera erradicada quando a percentagem de pessoas infetadas for inferior a 1%. b) Por questes de sade pblica e de acordo com a Organizao Mundial de Sade, este tipo de epidemia configura uma situao muito grave quando afeta uma populao em mais de 60% por um perodo superior a 24 horas. Na resoluo desta questo deves utilizar as capacidades grficas da tua calculadora e enriquecer a tua composio com o traado de um ou mais grficos. No obrigatria a determinao analtica de valores que consideres indispensveis, desde que os apresentes com uma aproximao razovel e indiques o processo que utilizaste recorrendo calculadora.

RESOLUO
49.1 H 32% de pessoas infetadas no momento da declarao da epidemia pois

Para determinar quando a percentagem de pessoas infetadas foi superior ou igual existente no momento da deteco da epidemia temos de resolver a inequao

Temos

visto que a funo exponencial de base superior a um estritamente crescente. Logo P (t ) 32 0,25t 2 + t + 5 5 t(0,25t 1) 0 t(0,25t 1) 0 0t 4 Assim, entre o momento inicial e o 4. dia a percentagem de pessoas infetadas foi superior ou igual existente no momento da declarao da epidemia. Passaram ento 4 24 = 96 horas em que a percentagem de pessoas infetadas foi superior ou igual existente no momento da deteco da epidemia. 49.2 Considerando, respetivamente, as janelas de visualizao [0,10][-1,70] e [0,3][50,70] 68
Recomendaes do GAVE

representaram-se graficamente as funes

cujos grficos se indicam a seguir

Considerando agora a janela de visualizao [0,10][0,2] representaram-se a mesma funo juntamente com a funo

cujos grficos se indicam a seguir. Criou-se ainda uma tabela de valores para a funo como se mostra na mesma figura:

Sabendo-se que

podemos concluir que

Recomendaes do GAVE

69

Reunindo toda esta informao podemos elaborar o grfico seguinte


P(%) 70 60 50 40 30 20 10 0 0 1,39 2,61 2 4 6 6,89 8 t(dias)

Do grfico conclui-se que a epidemia foi erradicada antes de se atingirem 7 dias, pelo que se veio a confirmar o prognstico do SNS quanto ao prazo de erradicao da epidemia. J quanto gravidade da situao no sucedeu o mesmo, pois veio a verificar-se que aproximadamente durante 29 horas (2,61 1,39 = 1,22 dias, ou seja, 1,22 24 = 29,28 horas) houve mais de 60% da populao afetada, pelo que, tendo sido ultrapassado o limiar referido, de acordo com a classificao da Organizao Mundial de Sade, este tipo de epidemia configura uma situao muito grave. Claro que este um modelo matemtico geral pelo que no pode dar por si s todas as indicaes sobre as medidas que deveriam ter sido tomadas no terreno, pelo que no h informao que permita avaliar as medidas tomadas.

TP
TAREFAS
50.
PROPOSTAS

Tarefa: Cultura de Amibas Os bilogos, para os seus estudos, realizam culturas de clulas. As amibas, seres unicelulares, reproduzem-se por bipartio, isto , cada uma divide-se em duas. Cada uma das novas amibas desenvolve-se, e quando chega ao momento prprio, divide-se novamente em duas, e assim sucessivamente. O nmero de amibas ir pois aumentar segundo a lei: 1 2 4 8 16 32 64 ...... 2t Esta lei, como adaptao realidade, tem defeitos. Enumera um ou dois. Nesta situao, se por hiptese as amibas se bipartissem de hora a hora e se no morressem, quantas amibas haveria ao fim de 15 horas? Mas, o tempo que decorre para cada partio no o mesmo para todas as amibas. Por outro lado, algumas amibas morrem antes de chegar fase da bipartio. Para descobrir o nmero

70

Recomendaes do GAVE

de amibas na cultura, necessrio fazer recontagens. Um bilogo contou as amibas que h em cada momento na sua cultura:
tempo (h) n. de amibas 0 4 1 6 2 9 3 13 4 20 5 30 6 46 7 68 8 103 9 154

Procura encontrar uma frmula que te permita obter, com o maior rigor possvel, o nmero de amibas em cada momento (utiliza a calculadora grfica e as curvas de regresso, procurando a mais adequada). 51. Considera a funo real de varivel real, assim deinida: t(x) = 1 + log (x 2 1). 51.1 51.2 51.3 51.4 Determina o domnio e os zeros da funo. Justiica que a funo no admite funo inversa. Resolve a condio t(x) < 0. Considera as funes, reais de varivel real, assim deinidas: f (x) = x + 1 g(x) = log x e h(x) = x2 1 Tendo em considerao que t(x) = (f o (g o h))(x) e ainda todo o estudo feito sobre as funes f, g e h, determina o contradomnio da funo t. Explica o teu raciocnio. 51.5 51.6 51.7 Mostra que a expresso algbrica da correspondncia (no funo) inversa da funo t x = 1 + 10y 1 e comprova o conjunto indicado na alnea anterior. Caracteriza j 1, funo inversa da funo t restrita a ]1, + [. Veriica na tua calculadora grica o representado a seguir:

51.8 51.9

Como explicas o observado confrontando-o com as respostas s alneas 51.3 e 51.4? Agora, utiliza um software de traado de gricos no computador para veriicar a resoluo deste exerccio.

Recomendaes do GAVE

71

52.

Utilizando uma calculadora grica a Ana descobriu que a equao log(x 2 ) = 2 log 3 tinha duas solues, que eram 3 e 3 . De seguida, resolveu algebricamente a equao seguindo os seguintes passos:

Onde est o erro? Justifica.

72

Recomendaes do GAVE

3.
1.

Testes de tempo limitado


T1

Teste 1 Probabilidades Escolha mltipla


45 minutos
Calculadora no autorizada

Uma caixa contm 6 bolas azuis e 4 bolas vermelhas. Duas bolas so tiradas da caixa, uma depois da outra, sem reposio. As aes descritas resultaro em acontecimentos que so (A) dependentes (B) independentes (C) complementares (D) mutuamente exclusivos
(adaptado de exames do Canad, estado de Alberta, 2002)

2.

De acordo com os resultados obtidos anteriormente, a probabilidade de uma equipa de basebol ganhar um jogo 4/5. A probabilidade de a equipa ganhar os prximos 2 jogos (A) 8/5 (B) 16/25 (C) 2/5 (D) 1/25
(adaptado de exames do Canad, estado de Alberta, 2002)

3.

O nmero de arranjos de 3 rapazes e 4 raparigas numa ila, se as raparigas tm de icar juntas, (A) (B) (C) (D)
(adaptado de exames do Canad, estado de Alberta, 2002)

Testes de tempo limitado

73

4.

Incluindo o Pedro e a Diana, uma determinada escola tem um Conselho Escolar com 10 membros. As probabilidades de 3 comisses possveis, cada uma contendo 4 membros desse Conselho, apresentada a seguir: Comisso 1: Pedro e Diana so ambos escolhidos

Comisso 2: S um de Pedro ou Diana so escolhidos

Comisso 3: Nem Pedro nem Diana so escolhidos

A probabilidade de Pedro ou Diana serem escolhidos : (A) (Probabilidade da Comisso 1) (Probabilidade da Comisso 2) (B) 1 - (Probabilidade da Comisso 3) (C) 1 - (Probabilidade da Comisso 1) (D) (Probabilidade da Comisso 3)
(adaptado de exames do Canad, estado de Alberta, 2001)

5.

Calcula

(A) 1 (B) 4 (C) 15 (D) 16


(adaptado de exames do Canad, estado de British Columbia, 2005)

74

Testes de tempo limitado

6.

No desenvolvimento do binmio (A) 4. termo (B) 5. termo (C) 6. termo (D) 7. termo

, o termo que contm

(adaptado de exames do Canad, estado de Manitoba, 2007)

7.

O diagrama mostra um espao de resultados com 13 acontecimentos igualmente provveis. Determina P(B).

(A)

(B)

(C)

(D)
(adaptado de exames do Canad, estado de British Columbia, 2006)

Testes de tempo limitado

75

8.

O diagrama abaixo mostra os gricos de duas distribuies normais com mdias desvios padro e , respetivamente.
y

X1 ~N(1,12 )

X2 ~N(2 ,22 )
x

Qual das seguintes afirmaes verdadeira? (A) (B) (C) (D) (E)
(adaptado de exames da Austrlia, estado de Victoria, 2003)

76

Testes de tempo limitado

T2

Teste 2 Probabilidades Escolha mltipla


45 minutos
Calculadora autorizada

1.

Na Estncia de Frias Luz do Sol, a probabilidade de chover em qualquer dos dias do ms de janeiro 0,1. A Glria vai passar 3 dias nessa Estncia de Frias em janeiro de 2013. Qual a probabilidade de que chova pelo menos num desses trs dias? (A) 0,001 (B) 0,271 (C) 0,3 (D) 0,729
(adaptado de exames da Austrlia, estado de New South Wales, 2000)

2.

Os nmeros 1 a 5 foram escritos em pedaos de papel separados e os papis colocados numa caixa. As letras A, B, C e D so cada uma escritas em papis diferentes e os papis so colocados numa caixa diferente. Jodi retira um pedao de papel de cada uma das caixas. O nmero de elementos do espao de resultados desta experincia aleatria (A) 51 (B) 20 (C) 9 (D) 2
(adaptado de exames do Canad, estado de Alberta, 2002)

3.

Os estudantes de um departamento de msica prepararam 6 coros contemporneos e 5 coros tradicionais. Para o concerto do departamento ser escolhido um programa em que apresentam 4 dos coros contemporneos e 3 dos coros tradicionais. Quantos programas diferentes podem ser apresentados, se a ordem dos coros no interessar? (A) 25 (B) 35 (C) 150 (D) 330
(adaptado de exames do Canad, estado de Alberta, 2002)

Testes de tempo limitado

77

4.

Foi efetuado um inqurito em que as pessoas tinham de colocar um X na caixa frente das atividades que as interessavam quando estavam de frias. Poderiam colocar tantos X em quantas atividades quisessem e poderiam deixar todas as caixas em branco. Ver paisagens Ir ao teatro Subir a uma montanha a p Praticar ski Visitar museus Praticar golfe Ir s compras

Antes de serem tabulados os resultados do inqurito foi preciso determinar quantas respostas diferentes se poderiam obter. Qual o nmero total de respostas diferentes possveis? (A) 28 (B) 128 (C) 5040 (D) 13700
(adaptado de exames do Canad, estado de Alberta, 2001)

5.

Uma festa de 18 pessoas dividida em 2 grupos diferentes consistindo de 11 pessoas e 7 pessoas. O nmero de modos diferentes de fazer isto : (A) (B) (C) (D)
(adaptado de exames do Canad, estado de Manitoba, 2007)

78

Testes de tempo limitado

6.

No desenvolvimento de (A) 9 (B) 10 (C) 36 (D) 45

, determina o coeiciente do termo contendo

(adaptado de exames do Canad, estado de British Columbia, 2006)

7.

Um investigador mdico mediu a temperatura corporal de 700 pessoas e descobriu que as temperaturas tinham uma distribuio normal com uma mdia de 36,8 graus Celsius e um desvio padro de 0,35 graus. O nmero de pessoas que se espera tenha uma temperatura corporal de 37,5 graus ou inferior (A) 16 (B) 68 (C) 490 (D) 684
(adaptado de exames do Canad, estado de Alberta, 2002)

8.

Dada uma curva normal com mdia 50 e desvio padro 10, determina o valor de (A) 0,0415 (B) 0,2333 (C) 0,2707 (D) 0,3075
(adaptado de exames do Canad, estado de British Columbia, 2006)

Testes de tempo limitado

79

T3

Teste 3 Probabilidades Itens de resposta aberta


45 minutos
Calculadora no autorizada

1.

Lanam-se dois dados cbicos equilibrados. Qual a probabilidade de um e um s dos dois nmeros obtidos ser um 5?
(adaptado de exames de Itlia, 2012)

2.

Um saco contm berlindes azuis e berlindes vermelhos na proporo de 2 para 3. Um berlinde selecionado ao acaso. Qual a probabilidade de o berlinde ser azul?
(adaptado de exames da Austrlia, estado de New South Wales, 2000)

3.

O nmero de combinaes de n objetos 4 a 4 igual ao nmero de combinaes dos mesmos objetos 3 a 3. Qual o valor de n?
(adaptado de exames de Itlia, 2011)

4.

Quantos so os nmeros distintos de 4 algarismos que possvel escrever usando os dgitos mpares?
(adaptado de exames de Itlia, 2008)

5.

A Renata vai vender o seu carro mas antes vai oicina fazer uma reviso. A probabilidade de o carro necessitar de uma mudana de leo de 0,3 e a probabilidade de o carro necessitar de um novo iltro de leo 0,5. A probabilidade de tanto o leo como o iltro do leo precisarem de ser mudados de 0,225. Se o leo precisar de ser mudado, qual a probabilidade de ser preciso novo iltro de leo?
(adaptado de exames da Nova Zelndia, 2006)

80

Testes de tempo limitado

T4

Teste 4 Probabilidades Itens de resposta aberta


90 minutos
Calculadora autorizada

1.

Queremos formar uma comisso de cinco pessoas, a ser escolhida entre 10 homens e 3 mulheres. Qual a probabilidade de a comisso ser constituda por 3 homens e 2 mulheres?
(adaptado de exames de Itlia, 2010)

2.

A Renata e o Estevo izeram uma sondagem na sua escola sobre as preferncias televisivas dos estudantes. 2.1 A Renata pediu a 150 estudantes escolhidos ao acaso que programas que tinham visto no dia anterior na televiso. O programa Shortland Street foi visto por 90 estudantes, 50 tinham visto o dolos e 30 tinham visto ambos. Qual a probabilidade de que um estudante escolhido ao acaso no tenha visto o Shortland Street nem o dolos? O Estevo fez uma sondagem a um outro grupo de estudantes igualmente escolhidos ao acaso sobre que tipos de programas tinham visto no im de semana anterior. Ele descobriu que 2/3 tinham visto desporto e que 4/9 tinham visto um ilme. Se 4/5 deles tinham visto pelo menos um programa de desporto ou ilme, qual a probabilidade de um estudante escolhido ao acaso ter visto tanto um programa de desporto como um ilme?
(adaptado de exames da Nova Zelndia, 2006)

2.2

3.

A Renata e o Estevo so ambos membros da direo da Associao de Estudantes que tem um total de 10 membros. Quando a direo da Associao de Estudantes foi apresentada escola durante uma assembleia, sentou-se no palco formando uma nica ila. Os lugares foram atribudos ao acaso. Qual a probabilidade de a Renata icar sentada na extremidade esquerda da ila e o Estevo icar sentado na extremidade direita da ila?
(adaptado de exames da Nova Zelndia, 2006)

4.

Os estudantes de uma Escola Bsica foram inquiridos sobre os seus almoos na escola. No dia do inqurito, 63% dos estudantes levaram para a escola o seu almoo, enquanto que o resto comprou o seu almoo, seja na escola seja em lojas prximas. Dos estudantes que foram para a escola com almoo trazido de casa, 84% dos seus almoos inclua fruta; apenas 47% dos lanches comprados inclua fruta. 4.1 4.2 Calcula a probabilidade de um estudante escolhido ao acaso dentre os inquiridos ter um almoo contendo fruta. Suponhamos que um estudante escolhido ao acaso dentre os que responderam ao inqurito tinha um almoo contendo fruta. Calcula a probabilidade de esse estudante ter comprado o almoo.

Testes de tempo limitado

81

4.3

Um inqurito no ano anterior tinha mostrado que 72% dos estudantes inquiridos tinha fruta no seu almoo. Dos estudantes que tinham fruta, descobriu-se que 56% tambm tinha sumo. Apenas 12% dos estudantes que no tinha fruta no seu almoo tinha sumo. Tambm se descobriu no inqurito que 60% dos estudantes que tinham tanto fruta como sumo nos seus almoos tinha comprado o seu almoo. Suponhamos que um aluno que respondeu ao inqurito do ano anterior foi escolhido ao acaso. Determina a probabilidade de o estudante ter um almoo comprado contendo fruta, sabendo que se descobriu que tinha sumo no seu almoo.
(adaptado de exames da Nova Zelndia, 2009)

5.

O Henrique s vezes vai para escola de carro e das outras vezes vai de autocarro. O Henrique s vezes leva o almoo de casa para a escola e outras vezes compra-o na escola. Num dia qualquer, a probabilidade do Henrique ir de carro para a escola 0,24 e a probabilidade de comprar o seu almoo 0,32. A probabilidade do Henrique ir de carro para a escola e comprar o seu almoo 0,0864. 5.1 Os acontecimentos Henrique vai de carro para a escola e Henrique compra o seu almoo so independentes? Justiica. Sempre que ele no tem de pagar o autocarro ou lhe do dinheiro para almoar, o Henrique coloca algum dinheiro no seu mealheiro. - Ele coloca 2 euros no seu mealheiro em cada dia em vai de carro para a escola e compra o seu almoo. - Ele coloca 1 euro no seu mealheiro em cada dia em vai de carro para a escola e no compra o seu almoo. - Ele coloca 50 cntimos no seu mealheiro em cada dia em vai de autocarro para a escola e compra o seu almoo. - Ele no coloca dinheiro no seu mealheiro em cada dia em vai de autocarro para a escola e no compra o seu almoo. Calcula o valor esperado de dinheiro que o Henrique coloca no seu mealheiro numa semana de aulas de 5 dias. Supe que o modo como o Henrique vai para a escola num dia no influencia o transporte noutro dia qualquer, e que o mesmo acontece com o almoo.
(adaptado de exames da Nova Zelndia, 2009)

5.2

82

Testes de tempo limitado

6.

No Dia da Diverso Matemtica, um dos jogos envolve a escolha aleatria de uma das seguintes cartas de uma caixa: M A T E M A T I C A

Regras do jogo: - Escolhe uma carta e ganhas o jogo! - Se no sair uma carta M recoloca essa carta na caixa e recomea o jogo. Escolhe uma carta M na segunda tentativa e ganhas o jogo! Se no sair uma carta dessas na segunda tentativa perdes o jogo! 6.1 6.2 Se for escolhida uma carta da caixa, qual a probabilidade de que seja uma carta M? Qual a probabilidade de ganhar este jogo?
(adaptado de exames da Austrlia, estado de New South Wales, 2000)

7.

Um jogo consiste em lanar dois dados regulares. O resultado a soma dos valores obtidos nos dois dados. O prmio dado de acordo com as seguintes regras: - A soma de 11 ou 12 d um prmio de 100 euros - A soma de 9 ou 10 d um prmio de 50 euros - A soma de 7 ou 8 d um prmio de 15 euros - A soma de 6 ou menos no d qualquer prmio. A varivel aleatria X representa o prmio num jogo. 7.1 7.2 Explica porque .

Completa o preenchimento da seguinte tabela: t 0 15 50 100

7.3 7.4

Determina a probabilidade de ganhar um prmio. Determina o valor mdio E(X).


(adaptado de exames da Dinamarca, 2007)

Testes de tempo limitado

83

8.

A Alice est a analisar o peso das ovelhas nascidas na sua quinta. 8.1 Ela descobre que o peso mdio 1,5 kg e o desvio padro 0,125 kg. Ela parte do princpio que os pesos tm uma distribuio normal. Qual a probabilidade de uma ovelha, escolhida ao acaso, pesar entre 1,5 kg e 1,7 kg? O Rafael calcula que a probabilidade de uma ovelha pesar entre 1,3 kg e 1,7 kg 0,9. Ele pensa que as ovelhas devem ser mais leves do que a Alice concluiu. Explica porque que no provvel que isto seja verdade. Qual a probabilidade de uma ovelha escolhida ao acaso da quinta da Alice pesar mais do que 1,8 kg? Uma ovelha considerada com peso a menos se pesar menos de 1,25 kg. As ovelhas com peso a menos raramente sobrevivem. Alice espera que lhe nasam 6400 ovelhas este ano. Quantas ovelhas deve ela esperar que morram devido a peso insuiciente?
(adaptado de exames da Nova Zelndia, 2011)

8.2

8.3 8.4

84

Testes de tempo limitado

T5

Teste 5 Probabilidades
90 minutos
Calculadora autorizada

I PARTE 1. Uma comisso numa escola consiste inclui 1 subdiretor, 2 professores e 3 estudantes. O nmero de comisses diferentes que podem ser formadas com 2 subdiretores, 5 professores e 9 estudantes (A) 20 160 (B) 8 008 (C) 1680 (D) 90
(adaptado de exames do Canad, estado de Alberta, 2001)

2.

Num baralho comum de 52 cartas, quantas mos diferentes de 4 cartas existem que contenham no mximo uma carta de copas? (A) 91 403 (B) 118 807 (C) 188 474 (D) 201 058
(adaptado de exames do Canad, estado de British Columbia, 2006)

3.

O David tira uma carta ao acaso de um baralho contendo 12 cartas vermelhas, 10 cartas amarelas, 5 cartas azuis, e 8 cartas verdes. Qual a probabilidade de que selecione uma carta azul ou uma carta vermelha? (A) 5/35 (B) 12/35 (C) 17/35 (D) 18/35
(adaptado de exames da Austrlia, estado de New South Wales, 2000)

Testes de tempo limitado

85

4.

O diretor de um colgio leu numa revista que os ps das mulheres estavam a aumentar. H alguns anos, a mdia do tamanho dos calados das mulheres era de 35,5 e, hoje, de 37,0. Embora no fosse uma informao cientica, ele icou curioso e fez uma pesquisa com as funcionrias do seu colgio, obtendo o quadro a seguir: Tamanho dos calados 39 38 37 36 35 Nmero de funcionrias 1 10 3 5 6

Escolhendo uma funcionria ao acaso e sabendo que ela tem calado maior que 36,0, a probabilidade de ela calar 38,0 (A) 1/3 (B) 1/5 (C) 2/5 (D) 5/7 (E) 5/14
(adaptado de exames do Brasil, 2010)

5.

A probabilidade de a Lisa ganhar um jogo

. Nenhum jogo termina num empate. Se ela

jogar dois jogos, qual a probabilidade de perder ambos? (A)

(B)

(C)

(D)
(adaptado de exames do Canad, estado de Manitoba, 2007)

86

Testes de tempo limitado

6.

Um cdigo postal no Canad constitudo por 3 letras e 3 dgitos ordenados de modo que tem primeiro uma letra, depois um dgito, depois uma letra e um dgito, e mais uma letra e um dgito. A primeira letra deve ser V, W ou X mas no h restries sobre as outras letras ou dgitos. Um exemplo de cdigo postal V0N 5Y2. Quantos cdigos postais diferentes so possveis? (A) 1 259 712 (B) 1 478 412 (C) 1 728 000 (D) 2 028 000
(adaptado de exames do Canad, estado de British Columbia, 2005)

7.

As primeiras 7 linhas do tringulo de Pascal so dadas a seguir: 1 1 1 1 1 1 1 O coeficiente de (A) 1080 (B) 540 (C) 10 (D) 540 (E) 1080
(adaptado de exames da Austrlia, estado de Victoria, 2003)

1 2 1 3 6 4 10 20 15 5 6 1 1 1 1 igual a

3 4

5 6 15

10

no desenvolvimento do binmio

Testes de tempo limitado

87

8.

O coeiciente do terceiro termo do desenvolvimento de (A) 21 (B) 35 (C) 84 (D) 140

(adaptado de exames do Canad, estado de British Columbia, 2005)

II PARTE 9. Suponhamos que um termo do desenvolvimento de , com b positivo,

Determina o valor de b, arredondado s dcimas.


(adaptado de exames do Canad, estado de Alberta, 2001)

10.

O Joo, a Amlia e o Frederico tentaram resolver o seguinte problema: Numa certa cidade, durante toda a vida de cada pessoa a probabilidade de ter diabetes 0,1 e a probabilidade de ter cancro 0,05. Qual a probabilidade de uma pessoa ter ou cancro ou ter diabetes durante toda a sua vida? Suponhamos que C o acontecimento ter cancro e D ter diabetes. Cada um dos estudantes props uma soluo diferente: Soluo do Joo: P(C e D) = 0,1 0,05 = 0,005 Soluo da Amlia: P(C ou D) = 0,1 + 0,05 = 0,15 Soluo do Frederico: P(C ou D) = 0,1 + 0,05 0,005 = 0,145 10.1 10.2 Qual dos estudantes tem a resoluo correta? Explica porque que as outras duas solues no esto corretas.
(adaptado de exames do Canad, estado de Nova Scotia, 2008)

11.

Se n > 3 e

esto em progresso aritmtica qual o valor de n?


(adaptado de exames de Itlia, 2010)

12.

mais provvel obter pelo menos um 6 lanando quatro vezes um dado cbico equilibrado ou obter pelo menos um 12 lanando vinte e quatro vezes dois dados?
(adaptado de exames de Itlia, 2007)

88

Testes de tempo limitado

Solues
1 Exerccios globais de 2. oportunidade
C1 CAPTULO 1
POSSVEL?

2.3.2 2.3.3 2.3.4

PROVVEL?

3. 3.1

Pratica
1. 1.1 1.2 1.3 1.4 1.5 2. 2.1 2.2 2.2.1 2.2.2 2.2.3 2.2.4 2.2.5 2.2.6 2.2.7 2.2.8 2.3 2.3.1 4. 4.1 M: ter sexo masculino ; F: ter sexo feminino quase impossvel pouco provvel bastante provvel bastante provvel quase certo 3.2 : sair o valete de copas : sair o 10 de copas : acontecimento impossvel : sair valete ou sair copas : sair copas ou o 10 de ouros : sair valete, ou sair o 10 de copas ou de ouros

M,M,M , M,M,F M,F,M , M,F,F S= F,M,M , F,M,F , F,F,M , F,F,F

( (

)( )(

)( )(

)( )(

) ,

4.2 4.3

7 Ocorrncias Diagrama de Venn: Seja A o acontecimento: pelo menos um dos filhos do sexo masculino.

Solues

89

A (M,M,M) (M,M,F) (F,F,F)

(L, L, L );(L, L,V , L );(L,V , L, L );(L,V ,V ,V ); S = (L, L,V ,V , L );(L,V ,V , L, L );(L,V , L,V , L ); (L, L,V ,V ,V );(L,V , L,V ,V );(L,V ,V , L,V )

7.
(M,F,M) (M,F,F) (F,M,M) (F,M,F) (F,F,M)

A: sair face par. : no sair face par. 1,1 , 1, 3 , 1, 5 , 1,7 , 1, 9 , A = 3, 5 , 3,7 , 3, 9 , 5, 5 , 5,7 , 5, 9 , 7,7 , 7, 9 , 9, 9

{ }{ }{ }{ }{ } { }{ }{ }{ }{ } { }{ }{ }{ }

Pensa e Resolve
8. 5.1
S=

5. Utilizando uma tabela de dupla entrada:

8.1 8.2 8.3 9. 9.1 9.2

A: sair a face com o nmero um B: sair um nmero natural inferior a dez C: sair o nmero dez

{1,1}, {1,2}, {1, 3}, {1, 4}, {1, 5}, {1, 6}, 2,2 , 2, 3 , 2, 4 , 2, 5 , 2, 6 , 3, 3 , { }{ }{ }{ }{ }{ } {3, 4}, {3, 5}, {3, 6}, {4, 4}, {4, 5}, {4, 6}, {5, 5}, {5, 6}, {6, 6}

5.2 5,1 , 5,2 , 5, 3 , 5, 4 , 5, 5 5, 6 , 5,7 , 5, 8 , 5, 9 5.3 8,1 , 8,2 , 8, 3 , 8, 4 , 8, 5 , 8, 6 8,7 , 8, 8 , 8, 9 , 9,1 , 9,2 , 9, 3 9, 4 , 9, 5 , 9, 6 , 9,7 , 9, 9 6. L: vence a equipa de Lamego V: vence a equipa de Viseu.

{ } { } { } { } { } , { }{ }{ }{ }
10. , ,

9.3 9.4 Considerem-se os acontecimentos, X: sai bola amarela Y: sai bola roxa 10.1 10.2 10.3 10.3.1

{ }{ }{ }{ }{ }{ } { }{ }{ }{ }{ }{ } { }{ }{ }{ }{ }

90

Solues

.] Utilizando a representao em diagramas de Venn podemos observar

A 10.3.2 Acontecimento certo: Acontecimento impossvel: A: sair face par B: sair face menor que 3 S

11.

, onde

, de

S hiptese de , temos , logo no um acontecimento impossvel. Na

Relete
12. 12.1 Falso. Basta considerar o acontecimento formado por todos os elementos do espao amostral. Falso. Basta considerarmos um acontecimento coincidente com o conjunto vazio. Verdadeira. impossvel ento .

Na

hiptese de , temos , ou seja um acontecimento impossvel. Conclumos assim que para que ser impossvel, os conjuntos tem de ser disjuntos, ou seja no tem elementos em comum, .

12.2

12.3 13. Se

C2 CAPTULO 2 PROBABILIDADES

por no se Por hiptese, tratarem de acontecimentos impossveis e de forma anloga, , por no se tratarem de acontecimentos certos. Ficamos ento com duas hipteses, ou ou .

Pratica
1.

2. [Sendo que, como


Solues

so diferentes, 91

3. 3.1

A: sair nmero par B: sair nmero impar

3.2 4. 4.1

8.3

Considerando os acontecimentos da alnea anterior, veriica-se que .

8.4

Considerem-se os acontecimentos A: sair nmero divisor de 5

4.2

B: sair nmero divisor de 6

4.3 5. 5.1 5.2 5.3 6. 7. 8. 0, 01 Consideremos, por exemplo, a experincia aleatria que consiste no lanamento de um dado equilibrado numerado de 1 a 6. 8.1 Considerem-se os acontecimentos 10. 10.1 9.

{ } { } B = {1,2, 3, 6} , B = {4, 5} ;
P(A) P(B ) A B = 1,2, 3, 4, 6, 8 ; P((A B ) (A B )) = 1

A = 1, 5 ; A = 2, 3, 4, 6 ;

11 12 1 12

10.2

Pensa e Resolve
11. 12. 11 12 Consideremos, por exemplo, a experincia aleatria que consiste no lanamento de um dado equilibrado numerado de 1 a 6. 12.1 Considerem-se os acontecimentos
Solues

A: sair um nmero primo B: sair um nmero menor que 3

8.2

Considerem-se os acontecimentos

92

A: sair um nmero par B: sair um nmero impar P(A) = P(B ) = 3 6 3 1 2 1 ; ; 15.

14.2 14.2.1 14.2.2

= =

14.2.3 Consideremos, por exemplo, os conjuntos A 1,2, 3 e B

6 3 P(A) = P(B )

4, 5, 6

12.2

Considerem-se os acontecimentos A: sair nmero primo B: sair nmero impar 16.

A e B so acontecimentos incompatveis

Relete
17. O jogo no equitativo pois o Joo tem mais probabilidade de ganhar que a Maria. O meu adversrio favorecido neste jogo.

18. 12.3 Considerando os acontecimentos da alnea anterior, tambm temos . 12.4 Considerem-se os acontecimentos 19.

19.1 19.2 19.3 19.4 19.5 20. 20.1

Falso. Falso. Falso. Verdadeira. Falso.

A: sair nmero divisor de 6 B: sair nmero divisor de 5 A = 1,2, 3, 6 ; A = 4, 5 ;

{ } { } B = {1, 5} ; B = {2, 3, 4, 6} ;

P(A) < P(B ) Feminino 13. Masculino 20.2

No CMA 171307 196304 367611

CMA 13354 15303 28 657

total 46,6% 53,4% 396 268

Consideremos os acontecimentos, A: um estudante de Cincias, Matemtica e Informtica CMA

14.

Considere-se o acontecimento 14.1

Solues

93

B: do sexo feminino 3.2 4. C: Estuda Cincias, Matemtica e Informtica CMA e do sexo feminino 4.1

4.2 5. D: do sexo masculino e no estuda Cincias, Matemtica e Informtica CMA 5.1

5.2 20.3 20.4 E: estudante CMA, ou do sexo masculino A e D so incompatveis pois so conjuntos disjuntos. 5.3

5.4

5.5

C3 CAPTULO 3 PROBABILIDADE
CONDICIONADA

5.6 6. 6.1 6.2 14 6 14 16 7

Pratica
1. 2. 3. 3.1 94

6.3 6.4 6.5 7.

Solues

7.1

16.1 16.2

7.2

17. 18.

8.

Relete
19. Surdo No surdo 0,5289 0,4671

9. 10. 10.1

Masculino Feminino 20. 21. 22. 10.2 23. Sim. No.

0,0021 0,0019

11. 12.

Nada se pode dizer. Nada se pode dizer.


DES

C4 CAPTULO 4 DISTRIBUIO
DE PROBABILIDA-

Pensa e Resolve
13. 13.1 13.2 13.3 13.4 14. 14.1 14.2 14.3 15. 16. 0,94 0,95 0,24 15,3% 1,1% 94,2%% 5,8%

Pratica
1. 1.1 1.2 1.3 2. 0 1 2

0,5002 3.

Solues

95

y 1,0

11.

0,8

O valor obtido para a pontuao de cada resposta certa 5.856, pelo que arredondado s unidades, cada resposta certa deve valor 6 pontos.

0,6

= 1

= 0,5

0,4

C5 CAPTULO 5 ANLISE COMBINATRIA

0,2

x 4 2 0 2 4

Pratica
1. 2. 6760000 1470000 2600 21; 42 120 120 6 720 4294967296

4. 5.

Mdia = 3,45 e Desvio Padro = 1,69

5.1 5.2 5.3

0,95221 0,999571 0,5

3. 4. 5. 6. 7. 8. 9.

Pensa e Resolve
6. 7. uma proposio verdadeira, dado que a distribuio normal centrada relativamente mdia. Como o 3 est mais afastado do 5 do que o 6 e como queremos saber qual a rea, abaixo da curva, que maior. Ento a rea para maior do que a rea para .

10. 10.1 10.2 44352

Pensa e Resolve
11. uma letra

8. 8.1 8.2 9. Mdia = 3 Desvio Padro = 3,6

12. 12.1 12.2 8,7 1027 0,000009

P = 0,3456 13. 14. 10.1 10.2 15. 16.

Relete
10.

907200 20160 5040 6


Solues

96

17. 720 18. 24

C6 CAPTULO 6 TRINGULO MIO DE NEWTON


DE

PASCAL

BIN-

Relete
19. 19.1 105 19.2 58905 19.3 10660 19.4 42840 19.5 45815 20. 20.1 0,0476 20.2 0,9988 20.3 0,00119 21.

Pratica
1.
1 1 6 5 15 10 20 10 15 5 6 1 1

2. 3.

10; 20; 8; 56; 126; 36; 120; 45

3.1 3.2 4.

Pensa e Resolve
5. 6. 23.1 23.2 n r + 1 7.
a = 1; b = 5; c = 10; d = 10; e = 5; f=1 3 e 14 7. termo ; 8. termo

22. 23.

24.

8. 9.

25.

26.

Relete
27. 10. 10.1

Solues

97

= = +

10.2
=

1.2
y 8 6 4 2

f (x ) = 2x

11. 12. No tem soluo.

h(x ) = f (x

1) = 2x

C7 CAPTULO 7 FUNO
EXPONENCIAL
6 4 2 2 4 6

Pratica
1.3 1.
y y

f (x ) = 2 =

f (x ) = 2x

10 8

10 8 6

6 4 2

4 2 6 4 2

k(x ) = |f (x )| = |2x | = 2x = f (x ) = =
x 0 2 4

1.4
=

A
y

1.1
=
y 4 2

= f (x ) = 2x = =

f (x ) = 2 =

L(x ) = 2 f (x ) = 2 2x = g(x ) = f ( x ) = 2
x

==

= 4
x

2 x 6 4 2 2 4 6

2 4

= = = =

= = =

+
Solues

98

1.5
M (x ) =

A
= 1
x

4.3
=2
x

1 f (x )

2 =

y 8 6

5.
= =

f (x ) = 2x

5.1 5.2

4 = 2

5.3
x

Pensa e Resolve
6.
=
T 100 80

2.

A 2.1
= g(x ) = 2
x

=
y 8 6 4 2

T (t ) = 20 + 60e =

0,11t

6.1

60 40 20 0

=
0 10 20 30 40 50 60 70

x 6 4 2 2 4 6

6.2 6.3

80 Aproximadamente 3,7 minutos Nos primeiros dois minutos No instante inicial =


= +

=
h(x ) = 1 x
3

6.4 6.5 6.6


x

2 5

A temperatura tende a igualar a temperatura ambiente, que de 20. O grico d-nos a= informao de que a reta de equao assntota do grico de T, o que conirma o que airmmos.

2.2 3. 4. 4.1 4.2


Solues

{9, 94, 0, 0,74}


= =

7.
=

8. 8.1
=

0 0
+

= =

= =

8.2 8.3

99
=

9. 9.1
= = = = = = = = =

ne. Se fosse tomado o doente nunca mais acor= daria. 10.4


=
5

=
x

9.2 9.3
=

4 3 2

N (t ) = A
x

100 97

Relete
10. 10.1
=

= P(t ) = A2
x

1 0 0 5

T (t ) = A

100 84
x 20 25 30

10

15

; ...

; 10.5 O Pentobombitone, pois permite adormecer com alguma facilidade e desaparece do sangue mais rpido = que os outros. 4 horas. No.
=

10.2
y 5 4 3 2 1 0 0 5 10 15 20 25 30
2
x

10.6 10.7
mg/L 8 6

T (t ) = A

100 84

10.3
y 8 6
x

0 0 50 1 hora 100 1 hora 150 200 1 hora 250 1 hora

t em minutos

M (t ) = A(1,15)x

4 2
x

N (t ) = A

100 97
x

A concentrao mxima nunca ultrapassar 8mg/l. Se considerarmos a sucesso das n horas = tero q1 = 4 0, 5 quantidades qn

P(t ) = A2

T (t ) = A
x 0 5 10 15 20 25 30

100 84

( ) ao fim de 1, 2, 3,
=

O medicamento que no real o Methohexito100


Solues

q2 = 4 0, 52 + 0, 5

) ) )
17. 16.

15.1 15.2

3,5 1,79

q3 = 4 0, 53 + 0, 52 + 0, 5

qn = 4 0, 5n + 0, 5n 1 + ... + 0, 5 Quando n tende para infinito tende para 4. 11. 11.1 11.2 11.3 12. 12.1 12.2 12.3 12.4 1% O pior momento ocorreu no 20. dia com uma percentagem de 54,6% A partir do 40. dia. 42,52% 1,124 10186 2,106 anos.

16.1 16.2

17.1

c(6) = 12 ln(6) + 200 221, 501 c(60) = 120 ln(60) + 200 691, 321

17.2

Pensa e Resolve
18.

19. 20. 20.1 ; x ]2, 68, +[

C7 CAPTULO 8 FUNO
LOGARTMICA

20.2 21. 22. 23.

Pratica
13. 13.1 13.2 13.3 14. 14.1 14.2 15. 4,25 14,25 0 23

23.1

23.2

23.3 24. 24.1 Estimativa: 8 ; Valor aproximado: 101

Solues

7,96 24.2 25. 25.1 25.2 25.3

29.3

29.4

2 Recomendaes do GAVE
C1 CAPTULO 1 RESOLUO
VIDA REAL DE PROBLEMAS DA

Relete
26. 27. 27.1 Falso! Com e vamos obter a igualdade numrica falsa: log 2(10) = 27.2 Falso! Com e vamos obter a igualdade numrica falsa: log 2(10) = 27.3 27.4 28. 28.1 28.2 29. 29.1 606 ln(2) ln(10) 3, 32 = 0, 30 log(2) log(10) 3, 32 = 0, 30 No!

6. 6.1 6.2 6.3 7. 8. 8.1 197


1 100

462 44 84

1/3; 1/3

17
9 100

3
9 10

Verdadeira! (Veja-se a pgina 100 do manual) Verdadeira! (Veja-se a pgina 100 do manual)

8.2 Olhando para a tarefa 3 o jogo parece ser do mesmo tipo pois h o dobro de raspadinhas e o dobro de prmios. Calculando o valor esperado obtemos E(X ) = 0,8 e assim o jogo favorvel aos jogadores pois sendo verdade que h o dobro de raspadinhas e o dobro de prmios, os prmios tm tambm o dobro do valor pelo que este jogo muito diferente do jogo da tarefa 3. 8.3 Poderia alterar o valor dos prmios, ou o nmero de prmios de modo que o valor esperado viesse igual a zero.

29.2

102

Solues

9. 9.1 9.2 10. 1 5 Tem de ser pelo menos n = 3 para haver


16.2 17. 17.1 17.2 17.3 18. 11.1 210 21 semanas 21 partidas 19. 19.1 28 18 20
0,5 6,5 96,5 18,5 6,5 0,5 2,5 2,5

22 500 euros. Aproximadamente 14 614 euros. 25%

pelo menos 120 apostas distintas pois 12 = 66 2 e 12 = 220


3

11.

18.1 f(x) = ln x se 1 x 6 e f(x) = ln (x + 12) se x < 6 11 18.2 f(6) = ln 6 1,79

11.2.1 11.2.2 12. 13. 13.1 13.2 13.3 14. 14.1 14.2 14.3 14.4 14.5 15. 16. 16.1 28 60

1 2
18,5 96,5
9

9 2
9

36 2
9

84 2
9

126 2
9

126 29

84 2
9

36 2
9

9 2
9

1 29

56; 48; 8. 8 12 24

19.2 O valor esperado positivo (favorvel ao jogador) e aproximadamente igual a 0,55 19.3 A probabilidade de acertar em 3 nmeros e 2 letras de 1 em 19.4 Seria igual pois havendo apenas 4 letras, acertar em 2 letras ou acertar em zero letras tem igual nmero de possibilidades.
1 1,5 2

6; 12; 24

19.5 O valor esperado negativo (favorvel aos organizadores) e aproximadamente igual a 0,197

Solues

103

19.6

Um exemplo de composio :

Tendo em considerao que h lucro esperado para o apostador no concurso da Escola Secundria Sidnio Pais, conforme o trabalho feito na resposta questo 2, tal significa que, em mdia, de cada vez que um jogador lana um disco no ser angariado qualquer financiamento para a viagem de finalistas como ainda haver lugar a prejuzo. No concurso da Escola Secundria Costa Lobo, o valor esperado para o apostador inferior ao valor que tem de pagar pela aposta, o que origina que em mdia, por cada aposta, haver lucro para os alunos. Assim, obviamente melhor, na perspectiva dos alunos, o concurso a realizar na Escola Secundria Costa Lobo, o Baixocentenas. alis o nico que pode permitir o cumprimento do objectivo para o que foi criado, financiar, pelo menos em parte, a viagem de finalistas. No entanto, tambm muito arriscado, uma vez que os alunos podero no garantir o pagamento dos prmios. Sabe-se, pela lei dos grandes nmeros, que a frequncia relativa dos acertos nas chaves premiadas estabilizar volta do valor de probabilidade, utilizado para calcular o valor esperado, se se realizar um nmero muito elevado de experincias, pelo que perfeitamente possvel haver, por exemplo, 5 ou 6 apostadores a obter o 1 prmio nas duas primeiras semanas, o que obrigar os alunos a terem um plafond de mais de 1000 euros para fazer face aos custos, e pior ser se nas duas semanas seguintes houver mais trs ou quatro vencedores. Por outro lado, em termos prticos h um problema difcil, ou mesmo impossvel, de ultrapassar. Para se chegar com segurana ao lucro pode ser necessrio fazer tantas experincias que no chegaro as apostas feitas na escola, mesmo que se prolongue o concurso por um ano inteiro, o que se justifica mais uma vez por causa da lei dos grandes nmeros. E mesmo que esse lucro venha a ocorrer, e considerando um pouco artificialmente que no haver grandes desvios entre os valores de probabilidade e as frequncias relativas, seriam necessrias 10000 apostas para se obter um lucro a rondar os 2000 euros, o que para uma viagem para todos os alunos da tur104

ma poderia no dar mais de 100 euros a cada um ((0,5 0,303) 10000 = 1970). Ser que o trabalho e as privaes para levar um concurso destes frente compensaro? Poderamos proceder a alteraes de modo a tornar o jogo lucrativo para os alunos da Escola Secundria Sidnio Pais, e para tentar aumentar o lucro da escola de Baixo. Relativamente Escola Secundria Costa Lobo poderia ser feito um aumento ligeiro do valor das apostas, mas de modo a no afastar os potenciais apostadores, ou ento diminuir o valor do prmio, mas a talvez diminua mais o interesse dos apostadores, mais do que o provocado por um ligeiro aumento no valor da aposta. Poderia tambm fazer-se campanhas publicitrias de modo a aumentar o interesse no jogo. Se se aumentar o valor da aposta para 0,75, o lucro esperado por aposta ser j de cerca de 0,447 (0,75 0,303), o que daria j 4470 ao fim de 10000 apostas, ou ento bastariam pouco mais de 4000 apostas (1970 : 0,447 4407) para dar sensivelmente o mesmo lucro que daria com os 0,50 por aposta ao fim das 10000 apostas. Relativamente ao concurso da Escola Secundria Sidnio Pais, bastaria aumentar a aposta para um valor superior ao valor esperado para o apostador, ou seja, 4 , por exemplo. No entanto cada aposta no daria para 1 cntimo de lucro (4 0,3997 = 0,003), o que faria com que demorasse mais do que o possvel para atingir os objectivos e, ainda por cima, baixar os valores dos prmios tornaria o jogo ainda menos apetecvel. Se a aposta fosse superior a 4 euros, considero que comearia a ser improvvel a adeso de apostadores, e mesmo a 4 euros no sei se no seria igualmente difcil. Mais vale adoptarem o concurso dos seus colegas da Escola Secundria Costa Lobo, ou melhor prepararem-se para um ano penoso e infrutfero. 20. 21. 22. 23. A B C A

Solues

C2 CAPTULO 2 - PROBLEMAS
MEROS REAIS QUE ENVOLVEM CLCULOS MAIS ELABORADOS NO CONJUNTO DOS N-

40.

C4 CAPTULO 4 - EXERCCIOS
QUE PRESSUPEM RACIOCNIOS DEMONSTRATIVOS

28. 28.1 43. 44. 45. 28.2 29. 29.1 46. 47.

C5 CAPTULO 5 - UTILIZAR
A CALCULADORA GRFICA PARA RESOLVER PROBLEMAS

29.2 30. 31. 32. 33. 33.1 33.2 33.3 33.4 34. 35. 36. 37. 38. 39. B B A C C D 28 84 1 2013 p=4 1,030 377 509 393 765 625

50. 51.

52. A condio dada tem domnio mas a condio tem apenas por domnio + e no o domnio da equao que pretendia resolver, pelo que a primeira equivalncia no vlida. Da no ter determinado a soluo negativa.

3 Testes de tempo limitado


T1 TESTE 1 PROBABILIDADES ESCOLHA
TIPLA ML-

1. 2. 3. 4.

A B A B

Solues

105

5. 6. 7. 8.

C C B D

2. 2.1 2.2 3. 4. 1/90 4/15 14/45

T2 TESTE 2 PROBABILIDADES ESCOLHA


TIPLA ML-

4.1 4.2 4.3

0,7031 0,24733 0,55385

1. 2. 3. 4. 5. 6. 7. 8.

B B C B B D D C

5. 5.1 Se tomarmos A: Henrique vai de carro para a escola e B: Henrique compra o seu almoo, a probabilidade de A se realizar 0,24 e a probabilidade de B se realizar 0,32. Temos 0,24 0,32 = 0,0768 mas sabemos que a probabilidade de A e B se realizarem 0,0864 um valor diferente. Logo os acontecimentos no so independentes. O valor esperado aproximadamente 2,2 euros.

5.2

T3 TESTE 3 PROBABILIDADES ITENS


POSTA ABERTA DE RES-

6. 6.1 6.2 7. 7.1 Porque existem 7 possibilidades de sada de uma soma de 9 ou 10 (6 + 3, 5 + 4, 4 + 5, 3 + 6, 6 + 4, 5 + 5, 4 + 6) num total de 36 resultados possveis no lanamento dos dois dados. 1/5 9/25

1. 2. 3. 4. 5.

5/18 2/5 n=7 120 0,45

7.2

T4 TESTE 4 PROBABILIDADES ITENS


POSTA ABERTA DE RES-

15

50

100

1.

360/1287

7.3

106

Solues

7.4

E(X)

aproximadamente

11. 12.

n=7 mais provvel obter pelo menos um 6 lanando quatro vezes um dado..

igual a 22,6 euros. 8. 8.1 8.2 0,4452 O Rafael no tem razo pois os valores 1,3 e 1,7 so simtricos em relao mdia prevista e sabemos que 0,95 a probabilidade, de acordo com a lei normal, de os valores se encontrarem entre 1,5 2 0,125 e 1,5 + 2 0,125 que so valores muito prximos dos estudados pelo Rafael. 0,02275 145 ou 146 ovelhas

8.3 8.4

T5 TESTE 5 PROBABILIDADES
1. 2. 3. 4. 5. 6. 7. 8. 9. 10. 10.1 10.2 O Frederico. A soluo do Joo no est correta porque ele calculou a probabilidade de C e D e no de C ou D. A soluo da Amlia no afasta a hiptese de C e D ocorrerem em simultneo. 107 C D C D C D E C 1,5

Solues

SNTESE
Um resumo do essencial Populao conjunto de elementos ou indivduos (no necessariamente pessoas) com caractersticas comuns. Varivel caracterstica comum a uma populao que assume valores diferentes de indivduo para indivduo. Experincia aleatria o processo que permite obter uma observao ou resultado tal que: antes da observao do fenmeno no se tem conhecimento suficiente para dizer qual dos resultados se vai verificar; possvel fazer um grande nmero de realizaes, independentes, da experincia; admite-se que possvel encontrar nmeros entre 0 e 1, que representam a frequncia relativa com que se verificam os resultados individuais de cada realizao da experincia. Espao de resultados S conjunto de resultados possveis associados a uma experincia aleatria. Acontecimento um subconjunto do espao de resultados S. Acontecimento elementar um acontecimento constitudo por um nico resultado, ou seja, um subconjunto do espao de resultados S formado por um nico elemento. Acontecimento certo um acontecimento igual ao espao de resultados S. Acontecimento impossvel um acontecimento igual ao conjunto vazio, representado por ou

{ }.
.

Acontecimento complementar ou contrrio do acontecimento A o acontecimento constitudo por todos os resultados de S que no esto em A. Representa-se por Ac ou

Acontecimento interseo dos acontecimentos A e B o acontecimento que se realiza . se e somente se A e B se realizam simultaneamente. Representa-se por Acontecimento unio dos acontecimentos A e B o acontecimento que se realiza se e somente se pelo menos um dos acontecimentos A ou B se realiza. Representa-se por . Acontecimento diferena dos acontecimentos A e B o acontecimento que se realiza ou se o acontecimentos A se realiza mas sem que B se realize. Representa-se por Definio frequencista de probabilidade de um acontecimento A o valor obtido para a frequncia relativa da realizao de A, num grande nmero de repeties da experincia aleatria. Definio de Laplace de Probabilidade: Se o espao de resultados S constitudo por um nmero finito n de elementos, todos eles igualmente possveis, define-se Probabilidade de um acontecimento A, e representa-se por P(A), a razo entre o nmero m de resultados favorveis a A (resultados que compem A) e o nmero n de resultados possveis (resultados que constituem S).

108

Sntese

Axiomtica da Probabilidade As noes primitivas so: espao de resultados; acontecimento. Considere-se um espao de resultados S, finito, e um conjunto W de acontecimentos (isto , subconjuntos de S) que satisfaam as seguintes condies: a) Se um acontecimento A est em W, ento o seu complementar b) Se dois acontecimentos A e B esto em W, ento a sua unio tambm est em W. tambm est em W.

A cada elemento associa-se um nmero que se chama Probabilidade de A e que se representa por P(A). Os axiomas a que P(A) satisfaz so: 1. axioma - A probabilidade de qualquer acontecimento sempre maior ou igual a zero: P(A) 0. 2. axioma - A probabilidade do acontecimento certo, S, 1: P(S) = 1. 3. axioma - Se dois acontecimentos so disjuntos, a probabilidade da sua unio igual soma das probabilidades de cada um: se ento

Propriedades das probabilidades 1. 2. A probabilidade do acontecimento impossvel zero, isto , Dado um acontecimento A, a probabilidade do acontecimento dada por 3. 4. 5. . ento . . . , contrrio de A,

Dados dois acontecimentos A e B, se Qualquer que seja o acontecimento A,

Se os acontecimentos A, B e C so disjuntos dois a dois ento

6.

Quaisquer que sejam os acontecimentos A e B:

7.

Quaisquer que sejam os acontecimentos A e B,

Probabilidade condicional: Dados dois acontecimentos A e B, com P(A) > 0, define-se a probabilidade condicional de B sabendo que A ocorreu e representa-se por P(B |A), ao quociente O acontecimento A independente do acontecimento B, com P(A) > 0 e P(B) > 0, se a probabilidade de A se verificar igual probabilidade condicional de A se realizar dado que B se realizou, isto P(A) = P(A|B)

Sntese

109

Uma distribuio de probabilidades da varivel aleatria discreta X ou funo massa de probabilidade de X um nmero finito de valores distintos x1, x2 ,..., xN da varivel aleatria X e um correspondente nmero de probabilidades p1 = P(X = x1 ), p2 = P(X = x 2 ),..., pN = P(X = xN ) tais que se tenha: a) 0 p 1,0 p 1,...,0 p 1 1 2 N b) p1 + p2 + ... + pN = 1 Valor mdio ou valor esperado da varivel aleatria X a seguinte quantidade:

Modelo Binomial varivel X que representa o nmero de sucessos em n observaes (provas) independentes umas das outras, em que em cada observao s se podem obter dois resultados possveis, sucesso ou insucesso, chama-se varivel aleatria com distribuio Binomial de parmetros n e p. O seu valor

para k = 0,1,2,, n, onde N(n,k) representa o nmero de vezes em que temos k sucessos e n k insucessos. Tem-se Modelo Normal As principais caractersticas da curva do modelo normal so: a) simtrica relativamente ao valor mdio da varivel, assumindo a o valor mximo; b) Quanto maior for o desvio padro mais achatada a curva; c) A rea compreendida entre a curva e o eixo dos XX igual a 1; P ( X + ) = 0,683 P ( 2 X + 2 ) = 0,954 P ( 3 X + 3 ) = 0,997 Princpio bsico da Anlise Combinatria para pares ordenados: O nmero total de pares ordenados que consegues formar quando para o primeiro elemento do par tens m hipteses e para o segundo elemento do par tens n hipteses, dado por m n. Arranjos Completos: Quando, de um conjunto com n elementos, escolhemos p elementos admitindo repeties, dizemos que estamos em presena de arranjos completos (com repetio). Temos: .

110

Sntese

Arranjos Simples: Dado um conjunto de n elementos o nmeros de arranjos simples (sem repetio) de p desses elementos igual ao produto dos p nmeros naturais consecutivos, por ordem decrescente, a partir de n. Temos:

Permutaes: Dado um conjunto de n elementos chamam-se permutaes dos n elementos aos arranjos desses elementos, n a n. Temos:

Combinaes: so um qualquer subconjunto de p elementos escolhidos de um conjunto com n elementos em que a ordem no interessa. Representam-se por se l combinaes de n elementos tomados p a p. Temos: ou ou ainda que

O tringulo de Pascal um tringulo de nmeros naturais em que os nmeros dos lados do tringulo so sempre iguais a 1 e cada elemento do tringulo (diferente de 1) se obtm somando os dois elementos imediatamente acima dele na linha de cima. Cada um dos nmeros do tringulo de Pascal pode ser representado por uma combinao em que o valor de cima o nmero da linha e o valor de baixo a posio na linha (comeando a contar as linhas e as posies no zero). Frmulas: 0! = 1, ,

Frmula do Binmio de Newton:

Propriedades da funo exponencial

de base a superior a um

O domnio , o contradomnio + , a funo contnua, estritamente crescente, e injetiva. , , , , , , , ,

Sntese

111

Propriedades da funo logartmica

de base a superior a um:

O Domnio da funo logartmica + e o Contradomnio . A funo logartmica contnua. Os grficos da funo exponencial e da funo logartmica so simtricos relativamente reta y = x, a bissetriz dos quadrantes mpares. , ., . Se x > 1 ento e se 0 < x < 1 ento

, desde que w seja positivo. positivos. desde que b seja positivo.

, desde que z e w sejam

O logaritmo do produto de dois nmeros reais (positivos) igual soma dos logaritmos dos fatores:

, Uma funo logartmica de base superior a um cresce para infinito mais lentamente do que qualquer potncia do seu argumento:

Frmula de mudana de base:

Quando a base for igual a 10, o logaritmo chama-se logaritmo decimal e designa-se apenas por log, quando a base for o nmero de Euler e, designa-se por logaritmo natural e escreve-se simplesmente ln.

112

Sntese

Jaime Carvalho e Silva


Professor Associado do Departamento de Matemtica da Faculdade de Cincias e Tecnologia da Universidade de Coimbra. Licenciado e Doutorado em Matemtica pela Universidade de Coimbra, estudou na Universidade de Paris 6. Foi professor visitante na Arizona State University (EUA) e Secretrio-Geral da Comisso Internacional de Instruo Matemtica (2009-2012). Professor h 36 anos na Universidade de Coimbra, leccionou disciplinas de Matemtica para Matemticos e Engenheiros, assim como da formao de professores de Matemtica e orientou Estgios Pedaggicos de Matemtica em sete escolas diferentes. Coordenador das Equipas Tcnicas que elaboraram os programa de Matemtica A, Matemtica B, MACS, Matemtica dos Cursos Profissionais e Matemtica das Escolas Artsticas. Consultor do GAVE desde a sua criao. Autor de Manuais Escolares do Ensino Bsico e do Ensino Secundrio tendo ganho o Prmio Sebastio e Silva da SPM para Manuais Escolares em 2005 e obtido uma Meno Honrosa em 2000.

Joaquim Pinto
Professor de Matemtica do Ensino Bsico e Secundrio h 20 anos, licenciado em Matemtica, ramo de formao Educacional, pelo Departamento de Matemtica da Faculdade de Cincias e Tecnologia da Universidade de Coimbra e Mestre em Ensino da Matemtica pelo Departamento de Matemtica da Faculdade de Cincias da Universidade do Porto. Desempenhou funes de Professor Acompanhante do Novo Programa de Matemtica do Ensino Secundrio e de Supervisor dos Exame de Matemtica A, continuando a ser classificador de Exames de Matemtica A. Orientou Estgio Pedaggico pelas Universidades de Aveiro e de Coimbra. Formador acreditado pelo Conselho Cientfico Pedaggico da Formao Contnua, nas reas: A43 Matemtica / Mtodos Quantitativos; C05 Didticas especficas (matemtica); e C15 Tecnologias Educativas (Informtica / Aplicaes da Informtica). Dinamizou vrias aes dentro dos referidos domnios.

Vladimiro Machado
Professor de Matemtica do Ensino Bsico e Secundrio h 30 anos, licenciado em Matemtica, ramo de formao Educacional, pelo Departamento de Matemtica da Faculdade de Cincias da Universidade do Porto e Mestre em Ensino da Matemtica pelo Departamento de Matemtica da Faculdade de Cincias da Universidade do Porto. Desempenhou funes de Professor Acompanhante do Novo Programa de Matemtica do Ensino Secundrio e de Supervisor dos Exame de Matemtica B. Desempenha as funes de Professor Acompanhante do Novo Programa de Matemtica do Ensino Bsico. Orientador de Estgio Pedaggico do Departamento de Matemtica da Faculdade de Cincias da Universidade do Porto. Formador acreditado pelo Conselho Cientfico Pedaggico da Formao Contnua, nas reas: A43 Matemtica / Mtodos Quantitativos; C05 Didticas especficas (Matemtica); e C15 Tecnologias Educativas (Informtica / Aplicaes da Informtica). Obra em 2 volumes
(No permitida a venda em separado)

DIO DE AUTOR

ISBN 978-989-97839-1-1

9 789899 783911

Anda mungkin juga menyukai